Library homepage

  • school Campus Bookshelves
  • menu_book Bookshelves
  • perm_media Learning Objects
  • login Login
  • how_to_reg Request Instructor Account
  • hub Instructor Commons

Margin Size

  • Download Page (PDF)
  • Download Full Book (PDF)
  • Periodic Table
  • Physics Constants
  • Scientific Calculator
  • Reference & Cite
  • Tools expand_more
  • Readability

selected template will load here

This action is not available.

Physics LibreTexts

5.1.1: Speeds of Different Types of Waves

  • Last updated
  • Save as PDF
  • Page ID 26167

  • Kyle Forinash and Wolfgang Christian

\( \newcommand{\vecs}[1]{\overset { \scriptstyle \rightharpoonup} {\mathbf{#1}} } \)

\( \newcommand{\vecd}[1]{\overset{-\!-\!\rightharpoonup}{\vphantom{a}\smash {#1}}} \)

\( \newcommand{\id}{\mathrm{id}}\) \( \newcommand{\Span}{\mathrm{span}}\)

( \newcommand{\kernel}{\mathrm{null}\,}\) \( \newcommand{\range}{\mathrm{range}\,}\)

\( \newcommand{\RealPart}{\mathrm{Re}}\) \( \newcommand{\ImaginaryPart}{\mathrm{Im}}\)

\( \newcommand{\Argument}{\mathrm{Arg}}\) \( \newcommand{\norm}[1]{\| #1 \|}\)

\( \newcommand{\inner}[2]{\langle #1, #2 \rangle}\)

\( \newcommand{\Span}{\mathrm{span}}\)

\( \newcommand{\id}{\mathrm{id}}\)

\( \newcommand{\kernel}{\mathrm{null}\,}\)

\( \newcommand{\range}{\mathrm{range}\,}\)

\( \newcommand{\RealPart}{\mathrm{Re}}\)

\( \newcommand{\ImaginaryPart}{\mathrm{Im}}\)

\( \newcommand{\Argument}{\mathrm{Arg}}\)

\( \newcommand{\norm}[1]{\| #1 \|}\)

\( \newcommand{\Span}{\mathrm{span}}\) \( \newcommand{\AA}{\unicode[.8,0]{x212B}}\)

\( \newcommand{\vectorA}[1]{\vec{#1}}      % arrow\)

\( \newcommand{\vectorAt}[1]{\vec{\text{#1}}}      % arrow\)

\( \newcommand{\vectorB}[1]{\overset { \scriptstyle \rightharpoonup} {\mathbf{#1}} } \)

\( \newcommand{\vectorC}[1]{\textbf{#1}} \)

\( \newcommand{\vectorD}[1]{\overrightarrow{#1}} \)

\( \newcommand{\vectorDt}[1]{\overrightarrow{\text{#1}}} \)

\( \newcommand{\vectE}[1]{\overset{-\!-\!\rightharpoonup}{\vphantom{a}\smash{\mathbf {#1}}}} \)

The speed of a wave is fixed by the type of wave and the physical properties of the medium in which it travels. An exception is electromagnetic waves which can travel through a vacuum. For most substances the material will vibrate obeying a Hooke's law force as a wave passes through it and the speed will not depend on frequency. Electromagnetic waves in a vacuum and waves traveling though a linear medium are termed linear waves and have constant speed. Examples:

  • For sound waves in a fluid (for example air or water) the speed is determined by \(v=(B/\rho )^{1/2}\) where \(B\) is the bulk modulus or compressibility of the fluid in newtons per meter squared and \(\rho\) is the density in kilograms per cubic meter.
  • For sound waves in a solid the speed is determined by \(v= (Y/\rho )^{1/2}\) where \(Y\) is Young's modulus or stiffness in Newtons per meter squared and \(\rho\) is the density in kilograms per meter cubed.
  • For waves on a string the speed is determined by \(v=(T/\mu )^{1/2}\) where \(T\) is the tension in the string in Newtons and \(\mu\) is the mass per length in kilograms per meter.
  • Although electromagnetic waves do not need a medium to travel (they can travel through a vacuum) their speed in a vacuum, \(c = (1/\mu _{o} ε_{o})^{1/2} = 3.0\times 10^{8}\text{ m/s}\) is governed by two physical constants, the permeability \(\mu_{o}\) and the permittivity, \(ε_{o}\) of free space (vacuum).

Table \(\PageIndex{1}\)

Here is a more comprehensive list of the speed of sound in various materials .

As we saw in the previous chapter, there is a relationship between the period, wavelength and speed of the wave. The period of a cork floating in the water is affected by how fast the wave passes (wave speed) and the distance between peaks (wavelength). The relationship between speed, period and wavelength of a sine wave is given by \(v=\lambda /T\) where wavelength and period for a sine wave were defined previously. This can also be written as \(v=\lambda f\) since frequency is the inverse of period and is true for all linear waves. Notice that, since wave speed is normally a fixed quantity the frequency and wavelength will be inversely proportion; higher frequencies mean shorter wavelengths.

Often it is easier to write \(ω = 2πf\) where \(\omega\) is the angular frequency in radians per second instead of having to write \(2\pi f\) everywhere. Likewise it is easier to write \(k=2\pi /\lambda \) where \(k\) is the wave number in radians per meter rather than having to write \(2\pi /\lambda\) a lot. (Note that \(k\) is not a spring constant here.) Using these new definitions the speed of a wave can also be written as \(v=f\lambda =\omega /k\).

If the medium is uniform the speed of a wave is fixed and does not change. There are circumstances where the speed of a particular wave does change, however. Notice that the speed of sound in air depends on the density of the air (mass per volume). But the density of air changes with temperature and humidity. So the speed of sound can be different on different days and in different locations. The temperature dependence of the speed of sound in air is given by \(v = 344 + 0.6 (T - 20)\) in meters per second where \(T\) is the temperature in Celsius (\(T\) here is temperature, not period). Notice that at room temperature (\(20^{\circ}\text{C}\)) sound travels at \(344\text{ m/s}\).

The speed of sound can also be affected by the movement of the medium in which it travels. For example, wind can carry sound waves further (i.e. faster) if the sound is traveling in the same direction or it can slow the sound down if the sound is traveling in a direction opposite to the wind direction.

Electromagnetic waves travel at \(\text{c} = 3.0\times 10^{8}\text{ m/s}\) in a vacuum but slow down when they pass through a medium (for example light passing from air to glass). This occurs because the material has a different value for the permittivity and/or permeability due to the interaction of the wave with the atoms of the material. The amount the speed changes is given by the index of refraction \(n=c/v\) where \(c\) is the speed of light in a vacuum and \(v\) is the speed in the medium. The frequency of the wave does not change when it slows down so, since \(v=\lambda f\), the wavelength of electromagnetic waves in a medium must be slightly smaller.

Video/audio examples:

  • What is the speed of sound in a vacuum? Buzzer in a bell jar . Why is there no sound when the air is removed from the jar?
  • Demonstration of speed of sound in different gasses . Why is there no sound when the air is removed from the jar?
  • These two videos demonstrate the Allasonic effect. The speed of sound is different in a liquid with air bubbles because the density is different. As the bubbles burst, the speed of sound changes, causing the frequency of sound waves in the liquid column to change, thus changing the pitch. Example: one , two . What do you hear in each case?
  • The Zube Tube is a toy that has a spring inside attached to two plastic cups on either end. Vibrations in the spring travel at different speeds so a sound starting at one end (for example a click when you shake the tube and the spring hits the cup) ends up changing pitch at the other end as the various frequencies arrive. In other words this is a nonlinear system. See if you can figure out from the video which frequencies travel faster, high frequencies or low.

Mini-lab on measuring the speed of sound .

Questions on Wave Speed:

\(f=1/T,\quad v=f\lambda ,\quad v=\omega /k,\quad k=2\pi /\lambda,\quad \omega =2\pi f,\quad y(x,t)=A\cos (kx-\omega t+\phi ),\quad v=\sqrt{B/Q}\)

  • Light travels at \(3.0\times 10^{8}\text{ m/s}\) but sound waves travel at about \(344\text{ m/s}\). What is the time delay for light and sound to arrive from a source that is \(10,000\text{ m}\) away (this can be used to get an approximate distance to a thunderstorm)?
  • What two mistakes are made in science fiction movies where you see and hear an explosion in space at the same time?
  • Consult the table for the speed of sound in various substances. If you have one ear in the water and one ear out while swimming in a lake and a bell is rung that is half way in the water some distance away, which ear hears the sound first?
  • At \(20\text{C}\) the speed of sound is \(344\text{ m/s}\). How far does sound travel in \(1\text{ s}\)? How far does sound travel in \(60\text{ s}\)?
  • Compare the last two answers with the distance traveled by light which has a speed of \(3.0\times 10^{8}\text{ m/s}\). Why do you see something happen before you hear it?
  • The speed of sound in water is \(1482\text{ m/s}\). How far does sound travel under water in \(1\text{ s}\)? How far does sound travel under water in \(60\text{ s}\)?
  • What happens to the speed of sound in air as temperature increases?
  • Using the equation for the speed of sound at different temperatures, what is the speed of sound on a hot day when the temperature is \(30^{\circ}\text{C}\)? Hint: \(v = 344\text{ m/s} + 0.6 (T - 20)\) where \(T\) is the temperature in Celsius.
  • Using the speed of sound at \(30^{\circ}\text{C}\) from the last question, recalculate the distance traveled for the cases in question four.
  • Suppose on a cold day the temperature is \(-10^{\circ}\text{C}\: (14^{\circ}\text{F}\)). You are playing in the marching band outside. How long does it take the sound from the band to reach the spectators if they are \(100\text{ m}\) away?
  • What is the difference in the speed of sound in air on a hot day (\(40^{\circ}\text{C}\)) and a cold day (\(0^{\circ}\text{C}\))?
  • What would an orchestra sound like if different instruments produced sounds that traveled at different speeds?
  • The speed of a wave is fixed by the medium it travels in so, for a given situation, is usually constant. What happens to the frequency of a wave if the wavelength is doubled?
  • What happens to the wavelength of a wave if the frequency is doubled and has the same speed?
  • Suppose a sound wave has a frequency \(200\text{ Hz}\). If the speed of sound is \(343\text{ m/s}\), what wavelength is this wave?
  • What factors determine the speed of sound in air?
  • Why do sound waves travel faster through liquids than air?
  • Why do sound waves travel faster through solids than liquids?
  • The speed of sound in a fluid is given by \(v=\sqrt{B/Q}\) where \(B\) is the Bulk Modulus (compressibility) and \(Q\) is the density. What happens to the speed if the density of the fluid increases?
  • What must be true about the compressibility, \(B\), of water versus air, given that sound travels faster in water and water is denser than air?
  • The speed of sound in a fluid is given by \(v=\sqrt{B/Q}\) where \(B\) is the Bulk Modulus (compressibility) and \(Q\) is the density. Can you think of a clever way to measure the Bulk Modulus of a fluid if you had an easy way to measure the speed of sound in a fluid? Explain.
  • The speed of sound on a string is given by \(v=\sqrt{T/\mu}\) where \(T\) is the tension in Newtons and \(\mu\) is the linear density (thickness) in \(\text{kg/m}\). You also know that \(v=f\lambda\). Give two ways of changing the frequency of vibration of a guitar string based on the knowledge of these two equations.
  • For the previous question, increasing the tension does what to the frequency? What does using a denser string do to the frequency?
  • The following graph is of a wave, frozen in time at \(t = 0\). The equation describing the wave is \(y(x,t)=A\cos (kx-\omega t+\phi )\). Sketch the effect of doubling the amplitude, \(A\).

clipboard_e5f764e637575a65b45b9f0eb09115dd2.png

Figure \(\PageIndex{1}\)

  • For the following graph of a wave, sketch the effect of doubling the wavelength.

clipboard_e37815ebeb535ef7dffe9296d845d3d06.png

Figure \(\PageIndex{2}\)

  • The mathematical description of a sine wave is given by \(y(x,t)=A\cos (kx-\omega t+\phi )\). Explain what each of the terms \((A, k, \omega, \phi )\) represent.

If you're seeing this message, it means we're having trouble loading external resources on our website.

If you're behind a web filter, please make sure that the domains *.kastatic.org and *.kasandbox.org are unblocked.

To log in and use all the features of Khan Academy, please enable JavaScript in your browser.

Physics library

Course: physics library   >   unit 8.

  • Production of sound
  • Sound Properties: Amplitude, period, frequency, wavelength

Speed of Sound

  • Relative speed of sound in solids, liquids, and gases
  • Mach numbers
  • Decibel Scale
  • Why do sounds get softer?
  • Ultrasound medical imaging

radio waves travel at speed of sound

Want to join the conversation?

  • Upvote Button navigates to signup page
  • Downvote Button navigates to signup page
  • Flag Button navigates to signup page

Good Answer

Video transcript

On the ground we have spread aluminium foil. Aluminium is a conductor, so this reflects the electric wave with a phase change of 180°, giving approximately zero electric field in the conductor. The receiving antenna measures the superposition of the incident and reflected waves. In the animation, the incident (electrical) wave is blue, the reflected wave is red, and the purple wave is the superposition: the total electric field at that point. The horizontal scale is arbitrary, the vertical scale is pretty accurate and time has been slowed down by a hundred million or so for us to see it.

Standing waves

Light, electromagnetism, time and space.

As mentioned above, the speed of electromagnetic radiation, c = √(1/ε 0 μ 0 ), appears as the characteristic speed in Maxwell's equations of electromagnetism .

It also appears in the theory of relativity, where it is the natural conversion between time and space. In space-time, the separation betwen two events (with separations of space Δx, Δy, Δz and in time Δt) is given by

Electromagnetic radiation travels through space without a medium. So, in retrospect, we can say that it is perhaps unsurprising that c is the natural relation between space and time. When Lamour, Lorentz, Fitzgerald and Einstein proposed this, however, this relation was much less obvious. We give an introduction to relativity in Einsteinlight .

Electromagnetic waves

The following link takes you to page where we measure the speed of light using laser light and time-of-flight. In the next, we use the same radio apparatus to investigate the polarisation of radio waves (and of light). The next one takes you back to the multimedia tutorial The Nature of Light ..

Youtube

  • TPC and eLearning
  • What's NEW at TPC?
  • Read Watch Interact
  • Practice Review Test
  • Teacher-Tools
  • Subscription Selection
  • Seat Calculator
  • Ad Free Account
  • Edit Profile Settings
  • Classes (Version 2)
  • Student Progress Edit
  • Task Properties
  • Export Student Progress
  • Task, Activities, and Scores
  • Metric Conversions Questions
  • Metric System Questions
  • Metric Estimation Questions
  • Significant Digits Questions
  • Proportional Reasoning
  • Acceleration
  • Distance-Displacement
  • Dots and Graphs
  • Graph That Motion
  • Match That Graph
  • Name That Motion
  • Motion Diagrams
  • Pos'n Time Graphs Numerical
  • Pos'n Time Graphs Conceptual
  • Up And Down - Questions
  • Balanced vs. Unbalanced Forces
  • Change of State
  • Force and Motion
  • Mass and Weight
  • Match That Free-Body Diagram
  • Net Force (and Acceleration) Ranking Tasks
  • Newton's Second Law
  • Normal Force Card Sort
  • Recognizing Forces
  • Air Resistance and Skydiving
  • Solve It! with Newton's Second Law
  • Which One Doesn't Belong?
  • Component Addition Questions
  • Head-to-Tail Vector Addition
  • Projectile Mathematics
  • Trajectory - Angle Launched Projectiles
  • Trajectory - Horizontally Launched Projectiles
  • Vector Addition
  • Vector Direction
  • Which One Doesn't Belong? Projectile Motion
  • Forces in 2-Dimensions
  • Being Impulsive About Momentum
  • Explosions - Law Breakers
  • Hit and Stick Collisions - Law Breakers
  • Case Studies: Impulse and Force
  • Impulse-Momentum Change Table
  • Keeping Track of Momentum - Hit and Stick
  • Keeping Track of Momentum - Hit and Bounce
  • What's Up (and Down) with KE and PE?
  • Energy Conservation Questions
  • Energy Dissipation Questions
  • Energy Ranking Tasks
  • LOL Charts (a.k.a., Energy Bar Charts)
  • Match That Bar Chart
  • Words and Charts Questions
  • Name That Energy
  • Stepping Up with PE and KE Questions
  • Case Studies - Circular Motion
  • Circular Logic
  • Forces and Free-Body Diagrams in Circular Motion
  • Gravitational Field Strength
  • Universal Gravitation
  • Angular Position and Displacement
  • Linear and Angular Velocity
  • Angular Acceleration
  • Rotational Inertia
  • Balanced vs. Unbalanced Torques
  • Getting a Handle on Torque
  • Torque-ing About Rotation
  • Properties of Matter
  • Fluid Pressure
  • Buoyant Force
  • Sinking, Floating, and Hanging
  • Pascal's Principle
  • Flow Velocity
  • Bernoulli's Principle
  • Balloon Interactions
  • Charge and Charging
  • Charge Interactions
  • Charging by Induction
  • Conductors and Insulators
  • Coulombs Law
  • Electric Field
  • Electric Field Intensity
  • Polarization
  • Case Studies: Electric Power
  • Know Your Potential
  • Light Bulb Anatomy
  • I = ∆V/R Equations as a Guide to Thinking
  • Parallel Circuits - ∆V = I•R Calculations
  • Resistance Ranking Tasks
  • Series Circuits - ∆V = I•R Calculations
  • Series vs. Parallel Circuits
  • Equivalent Resistance
  • Period and Frequency of a Pendulum
  • Pendulum Motion: Velocity and Force
  • Energy of a Pendulum
  • Period and Frequency of a Mass on a Spring
  • Horizontal Springs: Velocity and Force
  • Vertical Springs: Velocity and Force
  • Energy of a Mass on a Spring
  • Decibel Scale
  • Frequency and Period
  • Closed-End Air Columns
  • Name That Harmonic: Strings
  • Rocking the Boat
  • Wave Basics
  • Matching Pairs: Wave Characteristics
  • Wave Interference
  • Waves - Case Studies
  • Color Addition and Subtraction
  • Color Filters
  • If This, Then That: Color Subtraction
  • Light Intensity
  • Color Pigments
  • Converging Lenses
  • Curved Mirror Images
  • Law of Reflection
  • Refraction and Lenses
  • Total Internal Reflection
  • Who Can See Who?
  • Formulas and Atom Counting
  • Atomic Models
  • Bond Polarity
  • Entropy Questions
  • Cell Voltage Questions
  • Heat of Formation Questions
  • Reduction Potential Questions
  • Oxidation States Questions
  • Measuring the Quantity of Heat
  • Hess's Law
  • Oxidation-Reduction Questions
  • Galvanic Cells Questions
  • Thermal Stoichiometry
  • Molecular Polarity
  • Quantum Mechanics
  • Balancing Chemical Equations
  • Bronsted-Lowry Model of Acids and Bases
  • Classification of Matter
  • Collision Model of Reaction Rates
  • Density Ranking Tasks
  • Dissociation Reactions
  • Complete Electron Configurations
  • Elemental Measures
  • Enthalpy Change Questions
  • Equilibrium Concept
  • Equilibrium Constant Expression
  • Equilibrium Calculations - Questions
  • Equilibrium ICE Table
  • Intermolecular Forces Questions
  • Ionic Bonding
  • Lewis Electron Dot Structures
  • Limiting Reactants
  • Line Spectra Questions
  • Mass Stoichiometry
  • Measurement and Numbers
  • Metals, Nonmetals, and Metalloids
  • Metric Estimations
  • Metric System
  • Molarity Ranking Tasks
  • Mole Conversions
  • Name That Element
  • Names to Formulas
  • Names to Formulas 2
  • Nuclear Decay
  • Particles, Words, and Formulas
  • Periodic Trends
  • Precipitation Reactions and Net Ionic Equations
  • Pressure Concepts
  • Pressure-Temperature Gas Law
  • Pressure-Volume Gas Law
  • Chemical Reaction Types
  • Significant Digits and Measurement
  • States Of Matter Exercise
  • Stoichiometry Law Breakers
  • Stoichiometry - Math Relationships
  • Subatomic Particles
  • Spontaneity and Driving Forces
  • Gibbs Free Energy
  • Volume-Temperature Gas Law
  • Acid-Base Properties
  • Energy and Chemical Reactions
  • Chemical and Physical Properties
  • Valence Shell Electron Pair Repulsion Theory
  • Writing Balanced Chemical Equations
  • Mission CG1
  • Mission CG10
  • Mission CG2
  • Mission CG3
  • Mission CG4
  • Mission CG5
  • Mission CG6
  • Mission CG7
  • Mission CG8
  • Mission CG9
  • Mission EC1
  • Mission EC10
  • Mission EC11
  • Mission EC12
  • Mission EC2
  • Mission EC3
  • Mission EC4
  • Mission EC5
  • Mission EC6
  • Mission EC7
  • Mission EC8
  • Mission EC9
  • Mission RL1
  • Mission RL2
  • Mission RL3
  • Mission RL4
  • Mission RL5
  • Mission RL6
  • Mission KG7
  • Mission RL8
  • Mission KG9
  • Mission RL10
  • Mission RL11
  • Mission RM1
  • Mission RM2
  • Mission RM3
  • Mission RM4
  • Mission RM5
  • Mission RM6
  • Mission RM8
  • Mission RM10
  • Mission LC1
  • Mission RM11
  • Mission LC2
  • Mission LC3
  • Mission LC4
  • Mission LC5
  • Mission LC6
  • Mission LC8
  • Mission SM1
  • Mission SM2
  • Mission SM3
  • Mission SM4
  • Mission SM5
  • Mission SM6
  • Mission SM8
  • Mission SM10
  • Mission KG10
  • Mission SM11
  • Mission KG2
  • Mission KG3
  • Mission KG4
  • Mission KG5
  • Mission KG6
  • Mission KG8
  • Mission KG11
  • Mission F2D1
  • Mission F2D2
  • Mission F2D3
  • Mission F2D4
  • Mission F2D5
  • Mission F2D6
  • Mission KC1
  • Mission KC2
  • Mission KC3
  • Mission KC4
  • Mission KC5
  • Mission KC6
  • Mission KC7
  • Mission KC8
  • Mission AAA
  • Mission SM9
  • Mission LC7
  • Mission LC9
  • Mission NL1
  • Mission NL2
  • Mission NL3
  • Mission NL4
  • Mission NL5
  • Mission NL6
  • Mission NL7
  • Mission NL8
  • Mission NL9
  • Mission NL10
  • Mission NL11
  • Mission NL12
  • Mission MC1
  • Mission MC10
  • Mission MC2
  • Mission MC3
  • Mission MC4
  • Mission MC5
  • Mission MC6
  • Mission MC7
  • Mission MC8
  • Mission MC9
  • Mission RM7
  • Mission RM9
  • Mission RL7
  • Mission RL9
  • Mission SM7
  • Mission SE1
  • Mission SE10
  • Mission SE11
  • Mission SE12
  • Mission SE2
  • Mission SE3
  • Mission SE4
  • Mission SE5
  • Mission SE6
  • Mission SE7
  • Mission SE8
  • Mission SE9
  • Mission VP1
  • Mission VP10
  • Mission VP2
  • Mission VP3
  • Mission VP4
  • Mission VP5
  • Mission VP6
  • Mission VP7
  • Mission VP8
  • Mission VP9
  • Mission WM1
  • Mission WM2
  • Mission WM3
  • Mission WM4
  • Mission WM5
  • Mission WM6
  • Mission WM7
  • Mission WM8
  • Mission WE1
  • Mission WE10
  • Mission WE2
  • Mission WE3
  • Mission WE4
  • Mission WE5
  • Mission WE6
  • Mission WE7
  • Mission WE8
  • Mission WE9
  • Vector Walk Interactive
  • Name That Motion Interactive
  • Kinematic Graphing 1 Concept Checker
  • Kinematic Graphing 2 Concept Checker
  • Graph That Motion Interactive
  • Two Stage Rocket Interactive
  • Rocket Sled Concept Checker
  • Force Concept Checker
  • Free-Body Diagrams Concept Checker
  • Free-Body Diagrams The Sequel Concept Checker
  • Skydiving Concept Checker
  • Elevator Ride Concept Checker
  • Vector Addition Concept Checker
  • Vector Walk in Two Dimensions Interactive
  • Name That Vector Interactive
  • River Boat Simulator Concept Checker
  • Projectile Simulator 2 Concept Checker
  • Projectile Simulator 3 Concept Checker
  • Hit the Target Interactive
  • Turd the Target 1 Interactive
  • Turd the Target 2 Interactive
  • Balance It Interactive
  • Go For The Gold Interactive
  • Egg Drop Concept Checker
  • Fish Catch Concept Checker
  • Exploding Carts Concept Checker
  • Collision Carts - Inelastic Collisions Concept Checker
  • Its All Uphill Concept Checker
  • Stopping Distance Concept Checker
  • Chart That Motion Interactive
  • Roller Coaster Model Concept Checker
  • Uniform Circular Motion Concept Checker
  • Horizontal Circle Simulation Concept Checker
  • Vertical Circle Simulation Concept Checker
  • Race Track Concept Checker
  • Gravitational Fields Concept Checker
  • Orbital Motion Concept Checker
  • Angular Acceleration Concept Checker
  • Balance Beam Concept Checker
  • Torque Balancer Concept Checker
  • Aluminum Can Polarization Concept Checker
  • Charging Concept Checker
  • Name That Charge Simulation
  • Coulomb's Law Concept Checker
  • Electric Field Lines Concept Checker
  • Put the Charge in the Goal Concept Checker
  • Circuit Builder Concept Checker (Series Circuits)
  • Circuit Builder Concept Checker (Parallel Circuits)
  • Circuit Builder Concept Checker (∆V-I-R)
  • Circuit Builder Concept Checker (Voltage Drop)
  • Equivalent Resistance Interactive
  • Pendulum Motion Simulation Concept Checker
  • Mass on a Spring Simulation Concept Checker
  • Particle Wave Simulation Concept Checker
  • Boundary Behavior Simulation Concept Checker
  • Slinky Wave Simulator Concept Checker
  • Simple Wave Simulator Concept Checker
  • Wave Addition Simulation Concept Checker
  • Standing Wave Maker Simulation Concept Checker
  • Color Addition Concept Checker
  • Painting With CMY Concept Checker
  • Stage Lighting Concept Checker
  • Filtering Away Concept Checker
  • InterferencePatterns Concept Checker
  • Young's Experiment Interactive
  • Plane Mirror Images Interactive
  • Who Can See Who Concept Checker
  • Optics Bench (Mirrors) Concept Checker
  • Name That Image (Mirrors) Interactive
  • Refraction Concept Checker
  • Total Internal Reflection Concept Checker
  • Optics Bench (Lenses) Concept Checker
  • Kinematics Preview
  • Velocity Time Graphs Preview
  • Moving Cart on an Inclined Plane Preview
  • Stopping Distance Preview
  • Cart, Bricks, and Bands Preview
  • Fan Cart Study Preview
  • Friction Preview
  • Coffee Filter Lab Preview
  • Friction, Speed, and Stopping Distance Preview
  • Up and Down Preview
  • Projectile Range Preview
  • Ballistics Preview
  • Juggling Preview
  • Marshmallow Launcher Preview
  • Air Bag Safety Preview
  • Colliding Carts Preview
  • Collisions Preview
  • Engineering Safer Helmets Preview
  • Push the Plow Preview
  • Its All Uphill Preview
  • Energy on an Incline Preview
  • Modeling Roller Coasters Preview
  • Hot Wheels Stopping Distance Preview
  • Ball Bat Collision Preview
  • Energy in Fields Preview
  • Weightlessness Training Preview
  • Roller Coaster Loops Preview
  • Universal Gravitation Preview
  • Keplers Laws Preview
  • Kepler's Third Law Preview
  • Charge Interactions Preview
  • Sticky Tape Experiments Preview
  • Wire Gauge Preview
  • Voltage, Current, and Resistance Preview
  • Light Bulb Resistance Preview
  • Series and Parallel Circuits Preview
  • Thermal Equilibrium Preview
  • Linear Expansion Preview
  • Heating Curves Preview
  • Electricity and Magnetism - Part 1 Preview
  • Electricity and Magnetism - Part 2 Preview
  • Vibrating Mass on a Spring Preview
  • Period of a Pendulum Preview
  • Wave Speed Preview
  • Slinky-Experiments Preview
  • Standing Waves in a Rope Preview
  • Sound as a Pressure Wave Preview
  • DeciBel Scale Preview
  • DeciBels, Phons, and Sones Preview
  • Sound of Music Preview
  • Shedding Light on Light Bulbs Preview
  • Models of Light Preview
  • Electromagnetic Radiation Preview
  • Electromagnetic Spectrum Preview
  • EM Wave Communication Preview
  • Digitized Data Preview
  • Light Intensity Preview
  • Concave Mirrors Preview
  • Object Image Relations Preview
  • Snells Law Preview
  • Reflection vs. Transmission Preview
  • Magnification Lab Preview
  • Reactivity Preview
  • Ions and the Periodic Table Preview
  • Periodic Trends Preview
  • Intermolecular Forces Preview
  • Melting Points and Boiling Points Preview
  • Reaction Rates Preview
  • Ammonia Factory Preview
  • Stoichiometry Preview
  • Nuclear Chemistry Preview
  • Gaining Teacher Access
  • Tasks and Classes
  • Tasks - Classic
  • Subscription
  • Subscription Locator
  • 1-D Kinematics
  • Newton's Laws
  • Vectors - Motion and Forces in Two Dimensions
  • Momentum and Its Conservation
  • Work and Energy
  • Circular Motion and Satellite Motion
  • Thermal Physics
  • Static Electricity
  • Electric Circuits
  • Vibrations and Waves
  • Sound Waves and Music
  • Light and Color
  • Reflection and Mirrors
  • About the Physics Interactives
  • Task Tracker
  • Usage Policy
  • Newtons Laws
  • Vectors and Projectiles
  • Forces in 2D
  • Momentum and Collisions
  • Circular and Satellite Motion
  • Balance and Rotation
  • Electromagnetism
  • Waves and Sound
  • Atomic Physics
  • Forces in Two Dimensions
  • Work, Energy, and Power
  • Circular Motion and Gravitation
  • Sound Waves
  • 1-Dimensional Kinematics
  • Circular, Satellite, and Rotational Motion
  • Einstein's Theory of Special Relativity
  • Waves, Sound and Light
  • QuickTime Movies
  • About the Concept Builders
  • Pricing For Schools
  • Directions for Version 2
  • Measurement and Units
  • Relationships and Graphs
  • Rotation and Balance
  • Vibrational Motion
  • Reflection and Refraction
  • Teacher Accounts
  • Task Tracker Directions
  • Kinematic Concepts
  • Kinematic Graphing
  • Wave Motion
  • Sound and Music
  • About CalcPad
  • 1D Kinematics
  • Vectors and Forces in 2D
  • Simple Harmonic Motion
  • Rotational Kinematics
  • Rotation and Torque
  • Rotational Dynamics
  • Electric Fields, Potential, and Capacitance
  • Transient RC Circuits
  • Light Waves
  • Units and Measurement
  • Stoichiometry
  • Molarity and Solutions
  • Thermal Chemistry
  • Acids and Bases
  • Kinetics and Equilibrium
  • Solution Equilibria
  • Oxidation-Reduction
  • Nuclear Chemistry
  • Newton's Laws of Motion
  • Work and Energy Packet
  • Static Electricity Review
  • NGSS Alignments
  • 1D-Kinematics
  • Projectiles
  • Circular Motion
  • Magnetism and Electromagnetism
  • Graphing Practice
  • About the ACT
  • ACT Preparation
  • For Teachers
  • Other Resources
  • Solutions Guide
  • Solutions Guide Digital Download
  • Motion in One Dimension
  • Work, Energy and Power
  • Algebra Based Physics
  • Other Tools
  • Frequently Asked Questions
  • Purchasing the Download
  • Purchasing the CD
  • Purchasing the Digital Download
  • About the NGSS Corner
  • NGSS Search
  • Force and Motion DCIs - High School
  • Energy DCIs - High School
  • Wave Applications DCIs - High School
  • Force and Motion PEs - High School
  • Energy PEs - High School
  • Wave Applications PEs - High School
  • Crosscutting Concepts
  • The Practices
  • Physics Topics
  • NGSS Corner: Activity List
  • NGSS Corner: Infographics
  • About the Toolkits
  • Position-Velocity-Acceleration
  • Position-Time Graphs
  • Velocity-Time Graphs
  • Newton's First Law
  • Newton's Second Law
  • Newton's Third Law
  • Terminal Velocity
  • Projectile Motion
  • Forces in 2 Dimensions
  • Impulse and Momentum Change
  • Momentum Conservation
  • Work-Energy Fundamentals
  • Work-Energy Relationship
  • Roller Coaster Physics
  • Satellite Motion
  • Electric Fields
  • Circuit Concepts
  • Series Circuits
  • Parallel Circuits
  • Describing-Waves
  • Wave Behavior Toolkit
  • Standing Wave Patterns
  • Resonating Air Columns
  • Wave Model of Light
  • Plane Mirrors
  • Curved Mirrors
  • Teacher Guide
  • Using Lab Notebooks
  • Current Electricity
  • Light Waves and Color
  • Reflection and Ray Model of Light
  • Refraction and Ray Model of Light
  • Classes (Legacy Version)
  • Teacher Resources
  • Subscriptions

radio waves travel at speed of sound

  • Newton's Laws
  • Einstein's Theory of Special Relativity
  • About Concept Checkers
  • School Pricing
  • Newton's Laws of Motion
  • Newton's First Law
  • Newton's Third Law
  • The Speed of Sound
  • Pitch and Frequency
  • Intensity and the Decibel Scale
  • The Human Ear

radio waves travel at speed of sound

Since the speed of a wave is defined as the distance that a point on a wave (such as a compression or a rarefaction) travels per unit of time, it is often expressed in units of meters/second (abbreviated m/s). In equation form, this is

The faster a sound wave travels, the more distance it will cover in the same period of time. If a sound wave were observed to travel a distance of 700 meters in 2 seconds, then the speed of the wave would be 350 m/s. A slower wave would cover less distance - perhaps 660 meters - in the same time period of 2 seconds and thus have a speed of 330 m/s. Faster waves cover more distance in the same period of time.

Factors Affecting Wave Speed

The speed of any wave depends upon the properties of the medium through which the wave is traveling. Typically there are two essential types of properties that affect wave speed - inertial properties and elastic properties. Elastic properties are those properties related to the tendency of a material to maintain its shape and not deform whenever a force or stress is applied to it. A material such as steel will experience a very small deformation of shape (and dimension) when a stress is applied to it. Steel is a rigid material with a high elasticity. On the other hand, a material such as a rubber band is highly flexible; when a force is applied to stretch the rubber band, it deforms or changes its shape readily. A small stress on the rubber band causes a large deformation. Steel is considered to be a stiff or rigid material, whereas a rubber band is considered a flexible material. At the particle level, a stiff or rigid material is characterized by atoms and/or molecules with strong attractions for each other. When a force is applied in an attempt to stretch or deform the material, its strong particle interactions prevent this deformation and help the material maintain its shape. Rigid materials such as steel are considered to have a high elasticity. (Elastic modulus is the technical term). The phase of matter has a tremendous impact upon the elastic properties of the medium. In general, solids have the strongest interactions between particles, followed by liquids and then gases. For this reason, longitudinal sound waves travel faster in solids than they do in liquids than they do in gases. Even though the inertial factor may favor gases, the elastic factor has a greater influence on the speed ( v ) of a wave, thus yielding this general pattern:

Inertial properties are those properties related to the material's tendency to be sluggish to changes in its state of motion. The density of a medium is an example of an inertial property . The greater the inertia (i.e., mass density) of individual particles of the medium, the less responsive they will be to the interactions between neighboring particles and the slower that the wave will be. As stated above, sound waves travel faster in solids than they do in liquids than they do in gases. However, within a single phase of matter, the inertial property of density tends to be the property that has a greatest impact upon the speed of sound. A sound wave will travel faster in a less dense material than a more dense material. Thus, a sound wave will travel nearly three times faster in Helium than it will in air. This is mostly due to the lower mass of Helium particles as compared to air particles.  

The Speed of Sound in Air

The speed of a sound wave in air depends upon the properties of the air, mostly the temperature, and to a lesser degree, the humidity. Humidity is the result of water vapor being present in air. Like any liquid, water has a tendency to evaporate. As it does, particles of gaseous water become mixed in the air. This additional matter will affect the mass density of the air (an inertial property). The temperature will affect the strength of the particle interactions (an elastic property). At normal atmospheric pressure, the temperature dependence of the speed of a sound wave through dry air is approximated by the following equation:

where T is the temperature of the air in degrees Celsius. Using this equation to determine the speed of a sound wave in air at a temperature of 20 degrees Celsius yields the following solution.

v = 331 m/s + (0.6 m/s/C)•(20 C)

v = 331 m/s + 12 m/s

v = 343 m/s

(The above equation relating the speed of a sound wave in air to the temperature provides reasonably accurate speed values for temperatures between 0 and 100 Celsius. The equation itself does not have any theoretical basis; it is simply the result of inspecting temperature-speed data for this temperature range. Other equations do exist that are based upon theoretical reasoning and provide accurate data for all temperatures. Nonetheless, the equation above will be sufficient for our use as introductory Physics students.)

Look It Up!

Using wave speed to determine distances.

At normal atmospheric pressure and a temperature of 20 degrees Celsius, a sound wave will travel at approximately 343 m/s; this is approximately equal to 750 miles/hour. While this speed may seem fast by human standards (the fastest humans can sprint at approximately 11 m/s and highway speeds are approximately 30 m/s), the speed of a sound wave is slow in comparison to the speed of a light wave. Light travels through air at a speed of approximately 300 000 000 m/s; this is nearly 900 000 times the speed of sound. For this reason, humans can observe a detectable time delay between the thunder and the lightning during a storm. The arrival of the light wave from the location of the lightning strike occurs in so little time that it is essentially negligible. Yet the arrival of the sound wave from the location of the lightning strike occurs much later. The time delay between the arrival of the light wave (lightning) and the arrival of the sound wave (thunder) allows a person to approximate his/her distance from the storm location. For instance if the thunder is heard 3 seconds after the lightning is seen, then sound (whose speed is approximated as 345 m/s) has traveled a distance of

If this value is converted to miles (divide by 1600 m/1 mi), then the storm is a distance of 0.65 miles away.

Another phenomenon related to the perception of time delays between two events is an echo . A person can often perceive a time delay between the production of a sound and the arrival of a reflection of that sound off a distant barrier. If you have ever made a holler within a canyon, perhaps you have heard an echo of your holler off a distant canyon wall. The time delay between the holler and the echo corresponds to the time for the holler to travel the round-trip distance to the canyon wall and back. A measurement of this time would allow a person to estimate the one-way distance to the canyon wall. For instance if an echo is heard 1.40 seconds after making the holler , then the distance to the canyon wall can be found as follows:

The canyon wall is 242 meters away. You might have noticed that the time of 0.70 seconds is used in the equation. Since the time delay corresponds to the time for the holler to travel the round-trip distance to the canyon wall and back, the one-way distance to the canyon wall corresponds to one-half the time delay.

While an echo is of relatively minimal importance to humans, echolocation is an essential trick of the trade for bats. Being a nocturnal creature, bats must use sound waves to navigate and hunt. They produce short bursts of ultrasonic sound waves that reflect off objects in their surroundings and return. Their detection of the time delay between the sending and receiving of the pulses allows a bat to approximate the distance to surrounding objects. Some bats, known as Doppler bats, are capable of detecting the speed and direction of any moving objects by monitoring the changes in frequency of the reflected pulses. These bats are utilizing the physics of the Doppler effect discussed in an earlier unit (and also to be discussed later in Lesson 3 ). This method of echolocation enables a bat to navigate and to hunt.

The Wave Equation Revisited

Like any wave, a sound wave has a speed that is mathematically related to the frequency and the wavelength of the wave. As discussed in a previous unit , the mathematical relationship between speed, frequency and wavelength is given by the following equation.

Using the symbols v , λ , and f , the equation can be rewritten as

Check Your Understanding

1. An automatic focus camera is able to focus on objects by use of an ultrasonic sound wave. The camera sends out sound waves that reflect off distant objects and return to the camera. A sensor detects the time it takes for the waves to return and then determines the distance an object is from the camera. If a sound wave (speed = 340 m/s) returns to the camera 0.150 seconds after leaving the camera, how far away is the object?

Answer = 25.5 m

The speed of the sound wave is 340 m/s. The distance can be found using d = v • t resulting in an answer of 25.5 m. Use 0.075 seconds for the time since 0.150 seconds refers to the round-trip distance.

2. On a hot summer day, a pesky little mosquito produced its warning sound near your ear. The sound is produced by the beating of its wings at a rate of about 600 wing beats per second.

a. What is the frequency in Hertz of the sound wave? b. Assuming the sound wave moves with a velocity of 350 m/s, what is the wavelength of the wave?

Part a Answer: 600 Hz (given)

Part b Answer: 0.583 meters

3. Doubling the frequency of a wave source doubles the speed of the waves.

a. True b. False

Doubling the frequency will halve the wavelength; speed is unaffected by the alteration in the frequency. The speed of a wave depends upon the properties of the medium.

4. Playing middle C on the piano keyboard produces a sound with a frequency of 256 Hz. Assuming the speed of sound in air is 345 m/s, determine the wavelength of the sound corresponding to the note of middle C.

 Answer: 1.35 meters (rounded)

Let λ = wavelength. Use v = f • λ where v = 345 m/s and f = 256 Hz. Rearrange the equation to the form of λ = v / f. Substitute and solve.

5. Most people can detect frequencies as high as 20 000 Hz. Assuming the speed of sound in air is 345 m/s, determine the wavelength of the sound corresponding to this upper range of audible hearing.

Answer: 0.0173 meters (rounded)

Let λ = wavelength. Use v = f • λ where v = 345 m/s and f = 20 000 Hz. Rearrange the equation to the form of λ = v / f. Substitute and solve.

6. An elephant produces a 10 Hz sound wave. Assuming the speed of sound in air is 345 m/s, determine the wavelength of this infrasonic sound wave.

Answer: 34.5 meters

Let λ = wavelength. Use v = f • λ where v = 345 m/s and f = 10 Hz. Rearrange the equation to the form of λ = v / f. Substitute and solve.

7. Determine the speed of sound on a cold winter day (T=3 degrees C).

Answer: 332.8 m/s

The speed of sound in air is dependent upon the temperature of air. The dependence is expressed by the equation:

v = 331 m/s + (0.6 m/s/C) • T

where T is the temperature in Celsius. Substitute and solve.

v = 331 m/s + (0.6 m/s/C) • 3 C v = 331 m/s + 1.8 m/s v = 332.8 m/s

8. Miles Tugo is camping in Glacier National Park. In the midst of a glacier canyon, he makes a loud holler. He hears an echo 1.22 seconds later. The air temperature is 20 degrees C. How far away are the canyon walls?

Answer = 209 m

The speed of the sound wave at this temperature is 343 m/s (using the equation described in the Tutorial). The distance can be found using d = v • t resulting in an answer of 343 m. Use 0.61 second for the time since 1.22 seconds refers to the round-trip distance.

9. Two sound waves are traveling through a container of unknown gas. Wave A has a wavelength of 1.2 m. Wave B has a wavelength of 3.6 m. The velocity of wave B must be __________ the velocity of wave A.

a. one-ninth b. one-third c. the same as d. three times larger than

The speed of a wave does not depend upon its wavelength, but rather upon the properties of the medium. The medium has not changed, so neither has the speed.

10. Two sound waves are traveling through a container of unknown gas. Wave A has a wavelength of 1.2 m. Wave B has a wavelength of 3.6 m. The frequency of wave B must be __________ the frequency of wave A.

Since Wave B has three times the wavelength of Wave A, it must have one-third the frequency. Frequency and wavelength are inversely related.

  • Interference and Beats

NASA Logo

Anatomy of an Electromagnetic Wave

Energy, a measure of the ability to do work, comes in many forms and can transform from one type to another. Examples of stored or potential energy include batteries and water behind a dam. Objects in motion are examples of kinetic energy. Charged particles—such as electrons and protons—create electromagnetic fields when they move, and these fields transport the type of energy we call electromagnetic radiation, or light.

A photograph of a drop of water leaving ripples in a pool.

What are Electromagnetic and Mechanical waves?

Mechanical waves and electromagnetic waves are two important ways that energy is transported in the world around us. Waves in water and sound waves in air are two examples of mechanical waves. Mechanical waves are caused by a disturbance or vibration in matter, whether solid, gas, liquid, or plasma. Matter that waves are traveling through is called a medium. Water waves are formed by vibrations in a liquid and sound waves are formed by vibrations in a gas (air). These mechanical waves travel through a medium by causing the molecules to bump into each other, like falling dominoes transferring energy from one to the next. Sound waves cannot travel in the vacuum of space because there is no medium to transmit these mechanical waves.

An illustration in 3 panels — the first panel shows a wave approaching an insect sitting on the surface of the water. Second panel shows the wave passing underneath the insect, the insect stays in the same place but moves up as the wave passes. Third panel shows that the insect did not move with the wave, instead the wave had passed by the insect.

ELECTROMAGNETIC WAVES

Electricity can be static, like the energy that can make your hair stand on end. Magnetism can also be static, as it is in a refrigerator magnet. A changing magnetic field will induce a changing electric field and vice-versa—the two are linked. These changing fields form electromagnetic waves. Electromagnetic waves differ from mechanical waves in that they do not require a medium to propagate. This means that electromagnetic waves can travel not only through air and solid materials, but also through the vacuum of space.

In the 1860's and 1870's, a Scottish scientist named James Clerk Maxwell developed a scientific theory to explain electromagnetic waves. He noticed that electrical fields and magnetic fields can couple together to form electromagnetic waves. He summarized this relationship between electricity and magnetism into what are now referred to as "Maxwell's Equations."

A diagram of an electric field shown as a sine wave with red arrows beneath the curves and a magnetic field shown as a sine wave with blue arrows perpendicular to the electric field.

Heinrich Hertz, a German physicist, applied Maxwell's theories to the production and reception of radio waves. The unit of frequency of a radio wave -- one cycle per second -- is named the hertz, in honor of Heinrich Hertz.

His experiment with radio waves solved two problems. First, he had demonstrated in the concrete, what Maxwell had only theorized — that the velocity of radio waves was equal to the velocity of light! This proved that radio waves were a form of light! Second, Hertz found out how to make the electric and magnetic fields detach themselves from wires and go free as Maxwell's waves — electromagnetic waves.

WAVES OR PARTICLES? YES!

Light is made of discrete packets of energy called photons. Photons carry momentum, have no mass, and travel at the speed of light. All light has both particle-like and wave-like properties. How an instrument is designed to sense the light influences which of these properties are observed. An instrument that diffracts light into a spectrum for analysis is an example of observing the wave-like property of light. The particle-like nature of light is observed by detectors used in digital cameras—individual photons liberate electrons that are used for the detection and storage of the image data.

POLARIZATION

One of the physical properties of light is that it can be polarized. Polarization is a measurement of the electromagnetic field's alignment. In the figure above, the electric field (in red) is vertically polarized. Think of a throwing a Frisbee at a picket fence. In one orientation it will pass through, in another it will be rejected. This is similar to how sunglasses are able to eliminate glare by absorbing the polarized portion of the light.

DESCRIBING ELECTROMAGNETIC ENERGY

The terms light, electromagnetic waves, and radiation all refer to the same physical phenomenon: electromagnetic energy. This energy can be described by frequency, wavelength, or energy. All three are related mathematically such that if you know one, you can calculate the other two. Radio and microwaves are usually described in terms of frequency (Hertz), infrared and visible light in terms of wavelength (meters), and x-rays and gamma rays in terms of energy (electron volts). This is a scientific convention that allows the convenient use of units that have numbers that are neither too large nor too small.

The number of crests that pass a given point within one second is described as the frequency of the wave. One wave—or cycle—per second is called a Hertz (Hz), after Heinrich Hertz who established the existence of radio waves. A wave with two cycles that pass a point in one second has a frequency of 2 Hz.

Diagram showing frequency as the measurement of the number of wave crests that pass a given point in a second. Wavelength is measured as the distance between two crests.

Electromagnetic waves have crests and troughs similar to those of ocean waves. The distance between crests is the wavelength. The shortest wavelengths are just fractions of the size of an atom, while the longest wavelengths scientists currently study can be larger than the diameter of our planet!

An illustration showing a jump rope with each end being held by a person. As the people move the jump rope up and down very fast – adding MORE energy – the more wave crests appear, thus shorter wavelengths. When the people move the jump rope up and down slower, there are fewer wave crests within the same distance, thus longer wavelengths.

An electromagnetic wave can also be described in terms of its energy—in units of measure called electron volts (eV). An electron volt is the amount of kinetic energy needed to move an electron through one volt potential. Moving along the spectrum from long to short wavelengths, energy increases as the wavelength shortens. Consider a jump rope with its ends being pulled up and down. More energy is needed to make the rope have more waves.

Next: Wave Behaviors

National Aeronautics and Space Administration, Science Mission Directorate. (2010). Anatomy of an Electromagnetic Wave. Retrieved [insert date - e.g. August 10, 2016] , from NASA Science website: http://science.nasa.gov/ems/02_anatomy

Science Mission Directorate. "Anatomy of an Electromagnetic Wave" NASA Science . 2010. National Aeronautics and Space Administration. [insert date - e.g. 10 Aug. 2016] http://science.nasa.gov/ems/02_anatomy

Discover More Topics From NASA

James Webb Space Telescope

The image is divided horizontally by an undulating line between a cloudscape forming a nebula along the bottom portion and a comparatively clear upper portion. Speckled across both portions is a starfield, showing innumerable stars of many sizes. The smallest of these are small, distant, and faint points of light. The largest of these appear larger, closer, brighter, and more fully resolved with 8-point diffraction spikes. The upper portion of the image is blueish, and has wispy translucent cloud-like streaks rising from the nebula below. The orangish cloudy formation in the bottom half varies in density and ranges from translucent to opaque. The stars vary in color, the majority of which have a blue or orange hue. The cloud-like structure of the nebula contains ridges, peaks, and valleys – an appearance very similar to a mountain range. Three long diffraction spikes from the top right edge of the image suggest the presence of a large star just out of view.

Perseverance Rover

radio waves travel at speed of sound

Parker Solar Probe

radio waves travel at speed of sound

  • Random article
  • Teaching guide
  • Privacy & cookies

radio waves travel at speed of sound

by Chris Woodford . Last updated: July 23, 2023.

Photo: Sound is energy we hear made by things that vibrate. Photo by William R. Goodwin courtesy of US Navy and Wikimedia Commons .

What is sound?

Photo: Sensing with sound: Light doesn't travel well through ocean water: over half the light falling on the sea surface is absorbed within the first meter of water; 100m down and only 1 percent of the surface light remains. That's largely why mighty creatures of the deep rely on sound for communication and navigation. Whales, famously, "talk" to one another across entire ocean basins, while dolphins use sound, like bats, for echolocation. Photo by Bill Thompson courtesy of US Fish and Wildlife Service .

Robert Boyle's classic experiment

Artwork: Robert Boyle's famous experiment with an alarm clock.

How sound travels

Artwork: Sound waves and ocean waves compared. Top: Sound waves are longitudinal waves: the air moves back and forth along the same line as the wave travels, making alternate patterns of compressions and rarefactions. Bottom: Ocean waves are transverse waves: the water moves back and forth at right angles to the line in which the wave travels.

The science of sound waves

Picture: Reflected sound is extremely useful for "seeing" underwater where light doesn't really travel—that's the basic idea behind sonar. Here's a side-scan sonar (reflected sound) image of a World War II boat wrecked on the seabed. Photo courtesy of U.S. National Oceanographic and Atmospheric Administration, US Navy, and Wikimedia Commons .

Whispering galleries and amphitheaters

Photos by Carol M. Highsmith: 1) The Capitol in Washington, DC has a whispering gallery inside its dome. Photo credit: The George F. Landegger Collection of District of Columbia Photographs in Carol M. Highsmith's America, Library of Congress , Prints and Photographs Division. 2) It's easy to hear people talking in the curved memorial amphitheater building at Arlington National Cemetery, Arlington, Virginia. Photo credit: Photographs in the Carol M. Highsmith Archive, Library of Congress , Prints and Photographs Division.

Measuring waves

Understanding amplitude and frequency, why instruments sound different, the speed of sound.

Photo: Breaking through the sound barrier creates a sonic boom. The mist you can see, which is called a condensation cloud, isn't necessarily caused by an aircraft flying supersonic: it can occur at lower speeds too. It happens because moist air condenses due to the shock waves created by the plane. You might expect the plane to compress the air as it slices through. But the shock waves it generates alternately expand and contract the air, producing both compressions and rarefactions. The rarefactions cause very low pressure and it's these that make moisture in the air condense, producing the cloud you see here. Photo by John Gay courtesy of US Navy and Wikimedia Commons .

Why does sound go faster in some things than in others?

Chart: Generally, sound travels faster in solids (right) than in liquids (middle) or gases (left)... but there are exceptions!

How to measure the speed of sound

Sound in practice, if you liked this article..., find out more, on this website.

  • Electric guitars
  • Speech synthesis
  • Synthesizers

On other sites

  • Explore Sound : A comprehensive educational site from the Acoustical Society of America, with activities for students of all ages.
  • Sound Waves : A great collection of interactive science lessons from the University of Salford, which explains what sound waves are and the different ways in which they behave.

Educational books for younger readers

  • Sound (Science in a Flash) by Georgia Amson-Bradshaw. Franklin Watts/Hachette, 2020. Simple facts, experiments, and quizzes fill this book; the visually exciting design will appeal to reluctant readers. Also for ages 7–9.
  • Sound by Angela Royston. Raintree, 2017. A basic introduction to sound and musical sounds, including simple activities. Ages 7–9.
  • Experimenting with Sound Science Projects by Robert Gardner. Enslow Publishers, 2013. A comprehensive 120-page introduction, running through the science of sound in some detail, with plenty of hands-on projects and activities (including welcome coverage of how to run controlled experiments using the scientific method). Ages 9–12.
  • Cool Science: Experiments with Sound and Hearing by Chris Woodford. Gareth Stevens Inc, 2010. One of my own books, this is a short introduction to sound through practical activities, for ages 9–12.
  • Adventures in Sound with Max Axiom, Super Scientist by Emily Sohn. Capstone, 2007. The original, graphic novel (comic book) format should appeal to reluctant readers. Ages 8–10.

Popular science

  • The Sound Book: The Science of the Sonic Wonders of the World by Trevor Cox. W. W. Norton, 2014. An entertaining tour through everyday sound science.

Academic books

  • Master Handbook of Acoustics by F. Alton Everest and Ken Pohlmann. McGraw-Hill Education, 2015. A comprehensive reference for undergraduates and sound-design professionals.
  • The Science of Sound by Thomas D. Rossing, Paul A. Wheeler, and F. Richard Moore. Pearson, 2013. One of the most popular general undergraduate texts.

Text copyright © Chris Woodford 2009, 2021. All rights reserved. Full copyright notice and terms of use .

Rate this page

Tell your friends, cite this page, more to explore on our website....

  • Get the book
  • Send feedback

What Is the Speed of Radio Waves? The Surprising Answer!

Last Updated on Jan 23 2023

a cellphone, tablet and laptop on desk

Similar to light , radio waves are a type of electromagnetic radiation. They are used in communications and are most commonly seen in televisions and audio broadcasts but may also be used to send signals to and from spacecraft and space stations. Although many people think of them as a form of soundwave because they are converted by receivers to create audio, radio waves are actually electromagnetic, which means that they are similar to and travel at the same speed as light.

Radio waves travel at 300,000 kilometers per second. They can only achieve this speed in a vacuum but are only fractionally slower in Earth’s atmosphere.

  • What Is the Speed of Radio Waves?

Radio waves are electromagnetic radiation like sound waves, microwaves, and X-rays. All of these types of radiation travel at the same speed, which is 300,000 kilometers per second. This means that radio waves could travel around the earth seven times in a single second. It would take 8 minutes for them to travel from Earth to the Sun, and 4 years to reach the nearest star.

  • How Far Can a Radio Wave Travel?

Radio waves, and all forms of electromagnetic radiation, dissipate in Earth’s atmosphere, which means that they will eventually stop. However, in the void of space, they will travel on forever so they have no limit to the distance they will travel.

  • Are Radio Waves Harmful?

Radiofrequency radiation, which is the type of radiation caused by radio waves, is considered non-ionizing radiation, which means that it does not remove electrons from an atom and does not cause cancer. However, if the body absorbs enough radiofrequency radiation, it can cause parts of the body to heat up, which may cause burns and other related injuries.

It is also theorized that some forms of non-ionizing radiation may cause damage or changes to the body’s cells that lead to cancer, so while they don’t directly cause cancer, it is possible that some of this radiation may indirectly lead to cancerous changes of the body’s cells.

Radio waves are not considered harmful at the levels that most people are exposed to them, although research continues into the effects of non-ionizing radiation in general.

  • Does Rain Affect Radio Waves?

Radio waves are, or can be, affected by rain . The waves are reflected, refracted, and essentially diverted by the rain. This can lead to a phenomenon called rain fade, which means that the radio wave signal fades over distance, and it can have a significant impact on the use of radio waves for communication and other purposes.

  • Final Thoughts

Radio waves are used to transmit data, including pictures and audio, but while many people think of radio waves as a type of sound wave because radio plays sounds, radio waves are actually a type of electromagnetic radiation, which means that they are in the same class as light. They even travel at the same speed of light, which is slightly slower than 300,000 kilometers per second. Radio waves can travel to the Sun in 8 minutes but can be affected by rain. They are not thought to cause cancer in humans or animals.

  • https://www.nasa.gov/pdf/583093main_Earth_Calling.pdf
  • https://en.wikipedia.org/wiki/Radio_wave
  • https://www.qrg.northwestern.edu/projects/vss/docs/communications/2-why-does-it-take-so-long.html
  • https://www.ametsoc.org/index.cfm/ams/policy/policy-memos/the-radio-frequency-spectrum-and-weather-water-and-climate-uses-and-challenges/
  • https://physics.stackexchange.com/questions/461024/can-there-be-old-radio-waves-broadcasted-years-ago-still-traveling-in-the-air
  • https://www.cancer.org/healthy/cancer-causes/radiation-exposure/radiofrequency-radiation.html

Featured Image Credit: Mariakray, Pixabay

Table of Contents

About the Author Robert Sparks

Robert’s obsession with all things optical started early in life, when his optician father would bring home prototypes for Robert to play with. Nowadays, Robert is dedicated to helping others find the right optics for their needs. His hobbies include astronomy, astrophysics, and model building. Originally from Newark, NJ, he resides in Santa Fe, New Mexico, where the nighttime skies are filled with glittering stars.

Related Articles:

What Is the Best Binocular Magnification for Hunting? Optical Features Explained

How to Clean a Refractor Telescope: Step-by-Step Guide

How to Clean a Telescope Eyepiece: Step-by-Step Guide

How to Clean a Rifle Scope: 8 Expert Tips

Monocular vs Telescope: Differences Explained (With Pictures)

What Is a Monocular Used For? 8 Common Functions

How to Clean a Telescope Mirror: 8 Expert Tips

Brightfield vs Phase Contrast Microscopy: The Differences Explained

Wave Speed Calculator

Table of contents

Omni's wave speed calculator allows you to determine the speed of waves from their frequency and wavelength.

Continue reading this article to learn what wave speed is and how to calculate it. You will also find an example of wave speed calculation using this tool.

What is wave speed?

As we know, a wave is a disturbance that propagates from its point of origin. For example, when you throw a rock into a pond, the ripples or water waves move on the surface of the water in the outward direction from where you dropped the rock.

Wave speed is the speed at which the wave propagates . We can also define it as the distance traveled by the wave in a given time interval.

How to calculate wave speed - Wave speed formula

To calculate the speed of waves, we will use the formula:

  • v v v - Speed of the wave (in units of m/s);
  • f f f - Frequency of the wave (in units of hertz);
  • T T T - Period of the wave (in units of seconds); and
  • λ \lambda λ - Wavelength (in units of meters).

As an example, let us calculate the speed of sound waves in a medium where a 1500-Hz frequency produces a wavelength of 0.221 m.

We are given, f = 1500  Hz f=1500\ \text{Hz} f = 1500   Hz and λ = 0.221  m \lambda = 0.221\ \text m λ = 0.221   m .

Using the equation for wave speed, we can calculate

  • Hence, the speed of the sound waves in the given medium is 331.5 m/s.

How to use the wave speed calculator

Now we will see how to calculate wave speed using our calculator. We will consider the same example as in the previous section:

Enter the wave frequency ( f = 1500  Hz f=1500\ \text{Hz} f = 1500   Hz ) and wavelength ( λ = 0.221  m \lambda = 0.221\ \text m λ = 0.221   m ) in the respective fields.

The tool will display the wave speed ( v = 331.5  m/s v = 331.5\ \text{m/s} v = 331.5   m/s ).

You can calculate the speed of waves by typing the wave period (or wave frequency) and wave number (or wavelength).

Other wave calculators

If you liked this tool, do check out our other calculators to determine the properties of waves:

  • Wavelength calculator ;
  • Wavenumber calculator ; and
  • Wave velocity calculator .

How do I calculate wave speed from frequency and wavelength?

To calculate wave speed from frequency and wavelength, follow the given instructions:

  • Multiply the wavelength in meters with frequency in hertz.
  • You will get the wave speed in m/s.
  • Congrats! You have calculated wave speed from frequency and wavelength.

What is the speed of electromagnetic waves in a vacuum?

3.0 × 10⁸ m/s . All electromagnetic waves, including radio waves, travel at the same speed in a vacuum.

What is the SI unit of wave speed?

The SI unit of wave speed is m/s . According to the formula for wave speed, wave speed = wavelength/time period . Since the SI unit of wavelength is meter (m) and that of time period is second (s), the SI unit of wave speed is m/s.

Wave frequency (f)

Wave period (T)

Wavelength (λ)

Wavenumber (1/λ)

Wave speed (v)

Why does it take so long for the radio waves to travel through space?

Actually, radio waves travel very quickly through space. Radio waves are a kind of electromagnetic radiation, and thus they move at the speed of light. The speed of light is a little less than 300,000 km per second. At that speed, a beam of light could go around the Earth at the equator more then 7 times in a second.

The reason that it takes so long for radio messages to travel in space is that space is mind-bogglingly big. The distances to be traveled are so great that even light or radio waves take a while getting there. It takes around eight minutes for radio waves to travel from the Earth to the Sun, and four years to get from here to the nearest star.

How long does it take for transmissions to get between DS1 and Earth? How often is DS1 in communication with Earth? What are radio waves?
How is lag dealt with? Why does the data transfer rate have to drop with distance? What kind of data is DS1 sending back? How do the instruments and sensors coordinate sending signals? How much data is DS1 able to transfer? What is electromagnetic radiation?
How do you make a radio wave?

SaveNetRadio

How fast do radio waves travel.

We use radio waves from television and cellular service to navigation and air traffic control. Still, we don’t often stop appreciating them and just how crazy fast they really are. So, how fast do radio waves travel anyway?

Topics Covered - Index

How Fast Do Radio Waves Travel Through Space?

How long does it take for a radio signal to reach pluto, how long does it take for a radio message to travel from earth to the moon and back, how long does it take for radio waves to travel to the sun, how fast are radio waves compared to other types, can we detect radio waves from an alien civilization, so really, how fast do radio waves travel.

How Fast Does Radio Waves Travel on Earth or Moon or Sun

Unimpeded, radio waves travel at the speed of light because they are part of the electromagnetic spectrum. In terms of miles, radio waves travel at approximately 186,000 miles per second or 300,000,000 meters per second.

If you’re a science lover or just curious about the technology that makes your life easier, you’ve come to the right place. In the sections below, we will break down how fast radio waves travel, whether they’re in space or here on earth.

We’ll also answer interesting questions like how long it takes for radio waves to reach the moon or pluto. So let’s just dive right in!

There is a common misconception that radio waves travel slower through space than they do through the air. The truth is that radio waves travel at the speed of light, even in space. It might seem like it’s taking them longer because space is so vast that even light and radio waves take considerable time to make their way across it. 

There are galaxies we will never be able to see because they are so far away from us that the speed of light waves can’t keep up with the expansion of the universe. The same, of course, would be true of any radio waves coming from a civilization outside the observable universe. 

To get some perspective on how vast the distances are that radio waves travel through space , let’s see how long it takes for them to travel from our friendly rock Earth to the dwarf planet Pluto.

Radio waves take about four and a half hours to travel from Earth to Pluto. That’s because the waves must travel about three billion miles before reaching their destination. 

Now let’s look at an object that’s a little closer. Our moon . The question is, how long does it take for a radio message to travel from the earth to the moon and back? 

Radio waves can travel to the moon and back at an average of about 2.56 seconds. Therefore if you sent radio waves on a journey to the moon and back, it would be the blink of an eye before they return.  They can make it quickly because the distance from Earth to the moon is only about 238,855 miles. When compared to the 92.5 million miles between Earth and the Sun, that’s nothing. 

You may be wondering, what about the sun then? How long does it take for radio waves to travel from the earth to the sun?

Radio waves take eight minutes to make their way from the earth to the sun. 

That may seem like a short period, but remember, these waves are traveling at the speed of light. This just goes to show how unbelievably big our solar system is, let alone the whole universe. 

To really get an idea of just how incredibly fast radio waves to travel, you just need to compare them to other kinds of waves like sound waves and light waves. 

Below we’ve listed two other types of waves and their speed compared to radio waves:

  • Sound waves : Radio waves are a form of electromagnetic wave. Sound waves on the other hand, are a form of mechanical waves. Mechanical waves are not nearly as fast as electromagnetic waves because they are not made of light. Therefore sound waves can only travel 1,100 feet per second. That’s a far cry from the speed of light. 
  • Light waves : Like radio waves, light waves are also a form of electromagnetic wave. As such, light waves also travel at the speed of light. The main difference between light waves and radio waves is their frequency. 

The only thing that technically moves faster than the speed of radio waves or light isn’t a wave at all. The only thing faster than the speed of light is the expansion of the universe itself. That’s why radio waves outside the observable universe will never actually reach us.

  • Who Invented Radio?
  • VHF vs. UHF
  • Build a 40’ Antenna
  • What is a Two-Way Radio?
  • What is a DMR Ham Radio?

Let’s end on a fun note. Because radio waves can travel so far, so quickly, it’s only natural to wonder if we could detect radio waves sent out by an alien civilization living somewhere else in the universe.

While it is possible for us to detect radio waves from an alien civilization, the following issues make it less probable that we will:

  • The vastness of space: It’s hard to even wrap your head around just how ridiculously big the universe we live in is. Every indication we have now suggests that intelligent life is relatively rare, so knowing where to point our satellites is like a shot in the dark.
  • Radio waves diffuse: The real challenge is that as radio waves travel, they become diffused and unreadable. Therefore, if the advanced civilization is just a little too far away, it would be much harder to distinguish and interpret the radio waves they send. 

There have been scientific projects like SETI (Search for Extraterrestrial Intelligence) that have aimed satellites at the sky in the hopes of detecting a signal. Sadly, every single thing they’ve detected that seemed like it could be from aliens has turned out not to be so far. Still, the future isn’t written, so maybe someday that will be successful.

The only thing faster than traveling radio waves is the expansion of the universe. That’s because radio waves actually travel at the speed of light or 186,000 miles per second. 

This means that radio waves could travel to the sun in about eight minutes and to Pluto in about four and a half hours. Considering the vast distances between us and those objects, we can definitively say radio waves travel quickly. 

Give a Comment

Save my name, email, and website in this browser for the next time I comment.

15.1 The Electromagnetic Spectrum

Section learning objectives.

By the end of this section, you will be able to do the following:

  • Define the electromagnetic spectrum, and describe it in terms of frequencies and wavelengths
  • Describe and explain the differences and similarities of each section of the electromagnetic spectrum and the applications of radiation from those sections

Teacher Support

The learning objectives in this section will help your students master the following standards

  • (A) examine and describe oscillatory motion and wave propagation in various types of media;
  • (B) investigate and analyze characteristics of waves, including velocity, frequency, amplitude, and wavelength, and calculate using the relationship between wave speed, frequency, and wavelength;
  • (C) compare characteristics and behaviors of transverse waves, including electromagnetic waves and the electromagnetic spectrum, and characteristics and behaviors of longitudinal waves, including sound waves; and
  • (F) describe the role of wave characteristics and behaviors in medical and industrial applications.

In addition, the High School Physics Laboratory Manual addresses content in this section in the lab titled: Light and Color, as well as the following standards:

  • (C) compare characteristics and behaviors of transverse waves, including electromagnetic waves and the electromagnetic spectrum, and characteristics and behaviors of longitudinal waves, including sound waves.
  • (B) compare and explain the emission spectra produced by various atoms.

Section Key Terms

[BL] Explain that the term spectrum refers to a physical property that has a broad range with values that are continuous in some cases and, in other cases, discrete. Ask for other examples of spectra, for example, sound, people’s heights, etc.

[OL] Ask students to name ways that sunlight affects Earth. Provide examples that students don’t name: photosynthesis, weather, climate, seasons, warming, etc. Discuss energy transformations that take place after light enters the atmosphere, such as transformations in food chains and ecosystems. Ask students if they can explain how the energy in fossil fuels was originally light energy.

Misconception Alert

The light we can see is called visible light. Dispel any misconceptions that visible light is somehow different from radiation we cannot see, except for frequency and wavelength. The fact that some radiation is visible has to do with how the eye functions, not with the radiation itself.

The Electromagnetic Spectrum

We generally take light for granted, but it is a truly amazing and mysterious form of energy. Think about it: Light travels to Earth across millions of kilometers of empty space. When it reaches us, it interacts with matter in various ways to generate almost all the energy needed to support life, provide heat, and cause weather patterns. Light is a form of electromagnetic radiation (EMR) . The term light usually refers to visible light , but this is not the only form of EMR. As we will see, visible light occupies a narrow band in a broad range of types of electromagnetic radiation.

[OL] Discuss electric, magnetic, and gravitational fields. Point out how these three fields are similar, and how they differ.

[AL] Describe vectors as having magnitude and direction, and explain that fields are vector quantities. In these cases, the fields are made up of forces acting in a direction.

Electromagnetic radiation is generated by a moving electric charge, that is, by an electric current. As you will see when you study electricity, an electric current generates both an electric field , E , and a magnetic field , B . These fields are perpendicular to each other. When the moving charge oscillates, as in an alternating current, an EM wave is propagated. Figure 15.2 shows how an electromagnetic wave moves away from the source—indicated by the ~ symbol.

[BL] Review wave properties: frequency, wavelength, and amplitude. Ask students to recall sound and water waves, and explain how they relate to these properties.

[OL] Explain that an important difference between EM waves and other waves is that they can travel across empty space.

[AL] Ask if students remember the differences between longitudinal and transverse waves. Give examples. Explain that waves carry energy, not matter.

Watch Physics

Electromagnetic waves and the electromagnetic spectrum.

This video, link below, is closely related to the following figure. If you have questions about EM wave properties, the EM spectrum, how waves propagate, or definitions of any of the related terms, the answers can be found in this video .

Grasp Check

In an electromagnetic wave, how are the magnetic field, the electric field, and the direction of propagation oriented to each other?

  • All three are parallel to each other and are along the x -axis.
  • All three are mutually perpendicular to each other.
  • The electric field and magnetic fields are parallel to each other and perpendicular to the direction of propagation.
  • The magnetic field and direction of propagation are parallel to each other along the y -axis and perpendicular to the electric field.

Direct students to use this video as a way of connecting to the information in the following two figures, as well as to the following table.

Virtual Physics

Radio waves and electromagnetic fields.

This simulation demonstrates wave propagation. The EM wave is propagated from the broadcast tower on the left, just as in Figure 15.2 . You can make the wave yourself or allow the animation to send it. When the wave reaches the antenna on the right, it causes an oscillating current. This is how radio and television signals are transmitted and received.

Where do radio waves fall on the electromagnetic spectrum?

  • Radio waves have the same wavelengths as visible light.
  • Radio waves fall on the high-frequency side of visible light.
  • Radio waves fall on the short-wavelength side of visible light.
  • Radio waves fall on the low-frequency side of visible light.

Connect the discussion from the previous video, in which the generation of an electromagnetic wave is described, to this application of transmission and reception of electromagnetic waves. In particular, point out how the reception of the radio wave is essentially identical to the method by which the wave is generated. Explain also that these electromagnetic waves are the carrier waves on which audio or visual signals—either analog or digital—are placed, so that they can be transmitted to receivers within a certain range of the broadcast antenna.

From your study of sound waves, recall these features that apply to all types of waves:

  • Wavelength —The distance between two wave crests or two wave troughs, expressed in various metric measures of distance
  • Frequency —The number of wave crests that pass a point per second, expressed in hertz (Hz or s –1 )
  • Amplitude : The height of the crest above the null point

As mentioned, electromagnetic radiation takes several forms. These forms are characterized by a range of frequencies. Because frequency is inversely proportional to wavelength, any form of EMR can also be represented by its range of wavelengths. Figure 15.3 shows the frequency and wavelength ranges of various types of EMR. With how many of these types are you familiar?

Take a few minutes to study the positions of the various types of radiation on the EM spectrum, above. The narrow band that is visible light extends from lower-frequency red light to higher-frequency violet light. Frequencies just below the visible are called infrared (below red) and those just above are ultraviolet (beyond violet). Radio waves , which overlap with the frequencies used for media broadcasts of TV and radio signals, occupy frequencies even lower than infrared (IR). The microwave radiation that you see on the diagram is the same radiation that is used in a microwave oven. What we feel as radiant heat is also a form of low-frequency EMR. The high-frequency radiation to the right of ultraviolet (UV) includes X-rays and gamma (γ) rays.

[BL] Notice that most harmful forms of EM radiation are on the high-frequency end of the spectrum.

[OL] Ask which forms of EM radiation students have heard about. Ask them to describe the types of radiation they remember, and correct any misconceptions. Discuss the difference between ionizing radiation and nonionizing radiation, and the difference between electromagnetic radiation and other types of radiation—alpha, beta, etc.

Heat waves, a type of infrared radiation, are basically no different from other EM waves. We feel them as heat because they have a frequency that interacts with our bodies in a way that transforms EM energy into thermal energy.

Boundless Physics

Maxwell’s Equations

The Scottish physicist James Clerk Maxwell (1831–1879) is regarded widely to have been the greatest theoretical physicist of the nineteenth century. Although he died young, Maxwell not only formulated a complete electromagnetic theory, represented by Maxwell’s equations , he also developed the kinetic theory of gases, and made significant contributions to the understanding of color vision and the nature of Saturn’s rings.

Maxwell brought together all the work that had been done by brilliant physicists, such as Ørsted, Coulomb, Ampere, Gauss, and Faraday, and added his own insights to develop the overarching theory of electromagnetism. Maxwell’s equations are paraphrased here in words because their mathematical content is beyond the level of this text. However, the equations illustrate how apparently simple mathematical statements can elegantly unite and express a multitude of concepts—why mathematics is the language of science.

  • Electric field lines originate on positive charges and terminate on negative charges. The electric field is defined as the force per unit charge on a test charge, and the strength of the force is related to the electric constant, ε 0 .
  • Magnetic field lines are continuous, having no beginning or end. No magnetic monopoles are known to exist. The strength of the magnetic force is related to the magnetic constant, μ 0 .
  • A changing magnetic field induces an electromotive force (emf) and, hence, an electric field. The direction of the emf opposes the change, changing direction of the magnetic field.
  • Magnetic fields are generated by moving charges or by changing electric fields.

Maxwell’s complete theory shows that electric and magnetic forces are not separate, but different manifestations of the same thing—the electromagnetic force. This classical unification of forces is one motivation for current attempts to unify the four basic forces in nature—the gravitational, electromagnetic, strong nuclear, and weak nuclear forces. The weak nuclear and electromagnetic forces have been unified, and further unification with the strong nuclear force is expected; but, the unification of the gravitational force with the other three has proven to be a real head-scratcher.

One final accomplishment of Maxwell was his development in 1855 of a process that could produce color photographic images. In 1861, he and photographer Thomas Sutton worked together on this process. The color image was achieved by projecting red, blue, and green light through black-and-white photographs of a tartan ribbon, each photo itself exposed in different-colored light. The final image was projected onto a screen (see Figure 15.4 ).

Features that encouraged mathematicians and physicists to accept Maxwell’s equations is that they are seen as being both elegant and—considering the difference between an electric charge and a magnetic dipole, which give rise to the respective fields—essentially symmetrical. When scientists are looking for an approach to developing a new theory, they usually begin with the simplest and most symmetrical explanations. An example of such symmetry is the fact that electrons and protons have equal and opposite charges. You can see the symmetry in the four statements, given above, that describe the equations. The first two statements show a similar treatment of electric and magnetic fields, and the last two describe how a magnetic field can generate an electric field, and vice versa.

From our present-day perspective, we can now see the significance of Maxwell’s equations. This was the first step in the quest to unify all natural forces under one theory. After Maxwell unified the electric and magnetic forces as the electromagnetic force, others unified this force with the weak nuclear force, and there is evidence that the strong nuclear force can be unified with the electroweak force. The only force that has resisted unification with the others is the gravitational force. A theory that would unify all forces is often referred as a grand unified theory or a theory of everything . The quest for such a theory is still underway.

  • According to Maxwell’s equations, electromagnetic force gives rise to electric force and magnetic force.
  • According to Maxwell’s equations, electric force and magnetic force are different manifestations of electromagnetic force.
  • According to Maxwell’s equations, electric force is the cause of electromagnetic force.
  • According to Maxwell’s equations, magnetic force is the cause of electromagnetic force.

Characteristics of Electromagnetic Radiation

All the EM waves mentioned above are basically the same form of radiation. They can all travel across empty space, and they all travel at the speed of light in a vacuum. The basic difference between types of radiation is their differing frequencies. Each frequency has an associated wavelength. As frequency increases across the spectrum, wavelength decreases. Energy also increases with frequency. Because of this, higher frequencies penetrate matter more readily. Some of the properties and uses of the various EM spectrum bands are listed in Table 15.1 .

[BL] Explain transparency and opacity. Discuss how some materials are transparent to certain frequencies but opaque to others. Ask students for examples of materials that can be penetrated by some EM frequencies but not by others. Ask for examples of materials that are transparent to visible light and materials that are opaque to visible light.

[OL] Ask students why a lead apron is laid across dental patients during dental X-rays. Explain that X-rays are at the high-energy end of the spectrum and that they are very penetrating. They are only stopped by very dense materials, such as lead.

[AL] Ask if students can explain Earth’s greenhouse effect in terms of the penetrating power of various frequencies of EM radiation. Explain that the atmosphere is more transparent to visible light than to heat waves. Visible light penetrates the atmosphere and warms Earth’s surface. The heated surface radiates heat waves, which are trapped partially by certain gases in the atmosphere.

The narrow band of visible light is a combination of the colors of the rainbow. Figure 15.5 shows the section of the EM spectrum that includes visible light. The frequencies corresponding to these wavelengths are 4.0 × 10 14  s −1 4.0 × 10 14  s −1 at the red end to 7.9 × 10 14  s −1 7.9 × 10 14  s −1 at the violet end. This is a very narrow range, considering that the EM spectrum spans about 20 orders of magnitude.

[BL] Review the primary and secondary colors of pigments. Note that this is subtractive color mixing.

[OL] Explain the difference between subtractive and additive color mixing. The colors on the subtractive color wheel are made by pigments that absorb all colors but one. Therefore, when these colors all overlap, all light is absorbed and the result is black. White light is a combination of all colors, so when all colors are added together on the additive color wheel, the result is white. Explain that cyan is a shade of blue and that magenta is a shade of red.

Tips For Success

Wavelengths of visible light are often given in nanometers, nm. One nm equals 10 −9 10 −9 m. For example, yellow light has a wavelength of about 600 nm, or 6 × 10 −7 6 × 10 −7 m.

As a child, you probably learned the color wheel, shown on the left in Figure 15.6 . It helps if you know what color results when you mix different colors of paint together. Mixing two of the primary pigment colors—magenta, yellow, or cyan—together results in a secondary color. For example, mixing cyan and yellow makes green. This is called subtractive color mixing. Mixing different colors of light together is quite different. The diagram on the right shows additive color mixing. In this case, the primary colors are red, green, and blue, and the secondary colors are cyan, magenta, and yellow. Mixing pigments and mixing light are different because materials absorb light by a different set of rules than does the perception of light by the eye. Notice that, when all colors are subtracted, the result is no color, or black. When all colors are added, the result is white light. We see the reverse of this when white sunlight is separated into the visible spectrum by a prism or by raindrops when a rainbow appears in the sky.

Color Vision

This video demonstrates additive color and color filters. Try all the settings except Photons .

  • A blue filter absorbs blue light, causing the observed light to be a combination of the other colors.
  • A blue filter absorbs the opposite color of light—orange, causing the observed light to be blue.
  • A blue filter permits only blue light to pass though, absorbing the other colors and leaving blue light for the observer.
  • A blue filter permits only the opposite color light—orange—to pass through, leaving orange light for the observer.

Have students adjust the different colored lights for the RGB bulb simulation, first with individual settings, then with combinations of two and three colors to see what colors result and are perceived. Similarly, with the Single Bulb simulation, have students note how different filter settings affect what colors are seen for light with different color components.

Links To Physics

Animal color perception.

The physics of color perception has interesting links to zoology. Other animals have very different views of the world than humans, especially with respect to which colors can be seen. Color is detected by cells in the eye called cones . Humans have three cones that are sensitive to three different ranges of electromagnetic wavelengths. They are called red, blue, and green cones, although these colors do not correspond exactly to the centers of the three ranges. The ranges of wavelengths that each cone detects are red, 500 to 700 nm; green, 450 to 630 nm; and blue, 400 to 500 nm.

Most primates also have three kinds of cones and see the world much as we do. Most mammals other than primates only have two cones and have a less colorful view of things. Dogs, for example see blue and yellow, but are color blind to red and green. You might think that simpler species, such as fish and insects, would have less sophisticated vision, but this is not the case. Many birds, reptiles, amphibians, and insects have four or five different cones in their eyes. These species don’t have a wider range of perceived colors, but they see more hues, or combinations of colors. Also, some animals, such as bees or rattlesnakes, see a range of colors that is as broad as ours, but shifted into the ultraviolet or infrared.

These differences in color perception are generally adaptations that help the animals survive. Colorful tropical birds and fish display some colors that are too subtle for us to see. These colors are believed to play a role in the species mating rituals. Figure 15.7 shows the colors visible and the color range of vision in humans, bees, and dogs.

The symbiotic relationship between plants and their pollinators—bees, birds, etc.—is related to color perception. Plants have evolved to have flowers with colors that bees can see easily, and bees can find those flowers easily to collect the nectar they need for survival.

The belief that bulls are enraged by seeing the color red is a misconception. What did you read in this Links to Physics that shows why this belief is incorrect?

  • Bulls are color-blind to every color in the spectrum of colors.
  • Bulls are color-blind to the blue colors in the spectrum of colors.
  • Bulls are color-blind to the red colors in the spectrum of colors.
  • Bulls are color-blind to the green colors in the spectrum of colors.

Humans have found uses for every part of the electromagnetic spectrum. We will take a look at the uses of each range of frequencies, beginning with visible light. Most of our uses of visible light are obvious; without it our interaction with our surroundings would be much different. We might forget that nearly all of our food depends on the photosynthesis process in plants, and that the energy for this process comes from the visible part of the spectrum. Without photosynthesis, we would also have almost no oxygen in the atmosphere.

[BL] Ask how different frequencies of EM radiation are applied. Name each frequency range, and ask the students to supply the application, for example, X-rays used in medical imaging.

[OL] Ask students if they know why low-frequency radiation generally has different uses than high-frequency radiation. Explain that it has to do with penetrating power, which is related to health hazards.

[AL] Mention the ranges of TV signals designated very high frequency (VHF) and ultrahigh frequency (UHF). Explain that these frequencies are only relatively high compared to radio broadcast frequencies. Their place in the whole EM spectrum is at the low end.

The low-frequency, infrared region of the spectrum has many applications in media broadcasting. Television, radio, cell phone, and remote-control devices all broadcast and/or receive signals with these wavelengths. AM and FM radio signals are both low-frequency radiation. They are in different regions of the spectrum, but that is not their basic difference. AM and FM are abbreviations for amplitude modulation and frequency modulation . Information in AM signals has the form of changes in amplitude of the radio waves; information in FM signals has the form of changes in wave frequency .

Another application of long-wavelength radiation is found in microwave ovens. These appliances cook or warm food by irradiating it with EM radiation in the microwave frequency range. Most kitchen microwaves use a frequency of 2.45 × 10 9 2.45 × 10 9 Hz. These waves have the right amount of energy to cause polar molecules, such as water, to rotate faster. Polar molecules are those that have a partial charge separation. The rotational energy of these molecules is given up to surrounding matter as heat. The first microwave ovens were called Radaranges because they were based on radar technology developed during World War II.

Radar uses radiation with wavelengths similar to those of microwaves to detect the location and speed of distant objects, such as airplanes, weather formations, and motor vehicles. Radar information is obtained by receiving and analyzing the echoes of microwaves reflected by an object. The speed of the object can be measured using the Doppler shift of the returning waves. This is the same effect you learned about when you studied sound waves. Like sound waves, EM waves are shifted to higher frequencies by an object moving toward an observer, and to lower frequencies by an object moving away from the observer. Astronomers use this same Doppler effect to measure the speed at which distant galaxies are moving away from us. In this case, the shift in frequency is called the red shift , because visible frequencies are shifted toward the lower-frequency, red end of the spectrum.

Exposure to any radiation with frequencies greater than those of visible light carries some health hazards. All types of radiation in this range are known to cause cell damage. The danger is related to the high energy and penetrating ability of these EM waves. The likelihood of being harmed by any of this radiation depends largely on the amount of exposure. Most people try to reduce exposure to UV radiation from sunlight by using sunscreen and protective clothing. Physicians still use X-rays to diagnose medical problems, but the intensity of the radiation used is extremely low. Figure 15.8 shows an X-ray image of a patient’s chest cavity.

One medical-imaging technique that involves no danger of exposure is magnetic resonance imaging (MRI). MRI is an important imaging and research tool in medicine, producing highly detailed two- and three-dimensional images. Radio waves are broadcast, absorbed, and reemitted in a resonance process that is sensitive to the density of nuclei, usually hydrogen nuclei—protons.

Check Your Understanding

Use these questions to assess student achievement of the section’s Learning Objectives. If students are struggling with a specific objective, these questions will help identify any gaps and direct students to the relevant content.

Identify the fields produced by a moving charged particle.

  • Both an electric field and a magnetic field will be produced.
  • Neither a magnetic field nor an electric field will be produced.
  • A magnetic field, but no electric field will be produced.
  • Only the electric field, but no magnetic field will be produced.
  • Visible light has higher frequencies and shorter wavelengths than X-rays.
  • Visible light has lower frequencies and shorter wavelengths than X-rays.
  • Visible light has higher frequencies and longer wavelengths than X-rays.
  • Visible light has lower frequencies and longer wavelengths than X-rays.
  • The wavelength increases.
  • The wavelength first increases and then decreases.
  • The wavelength first decreases and then increases.
  • The wavelength decreases.
  • X-rays have higher penetrating energy than radio waves.
  • X-rays have lower penetrating energy than radio waves.
  • X-rays have a lower frequency range than radio waves.
  • X-rays have longer wavelengths than radio waves.
  • both an electric field and a magnetic field
  • neither a magnetic field nor an electric field
  • only a magnetic field, but no electric field
  • only an electric field, but no magnetic field

As an Amazon Associate we earn from qualifying purchases.

This book may not be used in the training of large language models or otherwise be ingested into large language models or generative AI offerings without OpenStax's permission.

Want to cite, share, or modify this book? This book uses the Creative Commons Attribution License and you must attribute Texas Education Agency (TEA). The original material is available at: https://www.texasgateway.org/book/tea-physics . Changes were made to the original material, including updates to art, structure, and other content updates.

Access for free at https://openstax.org/books/physics/pages/1-introduction
  • Authors: Paul Peter Urone, Roger Hinrichs
  • Publisher/website: OpenStax
  • Book title: Physics
  • Publication date: Mar 26, 2020
  • Location: Houston, Texas
  • Book URL: https://openstax.org/books/physics/pages/1-introduction
  • Section URL: https://openstax.org/books/physics/pages/15-1-the-electromagnetic-spectrum

© Jan 19, 2024 Texas Education Agency (TEA). The OpenStax name, OpenStax logo, OpenStax book covers, OpenStax CNX name, and OpenStax CNX logo are not subject to the Creative Commons license and may not be reproduced without the prior and express written consent of Rice University.

  • Skip to main content
  • Keyboard shortcuts for audio player

The huge solar storm is keeping power grid and satellite operators on edge

Geoff Brumfiel, photographed for NPR, 17 January 2019, in Washington DC.

Geoff Brumfiel

Willem Marx

radio waves travel at speed of sound

NASA's Solar Dynamics Observatory captured this image of solar flares early Saturday afternoon. The National Oceanic and Atmospheric Administration says there have been measurable effects and impacts from the geomagnetic storm. Solar Dynamics Observatory hide caption

NASA's Solar Dynamics Observatory captured this image of solar flares early Saturday afternoon. The National Oceanic and Atmospheric Administration says there have been measurable effects and impacts from the geomagnetic storm.

Planet Earth is getting rocked by the biggest solar storm in decades – and the potential effects have those people in charge of power grids, communications systems and satellites on edge.

The National Oceanic and Atmospheric Administration says there have been measurable effects and impacts from the geomagnetic storm that has been visible as aurora across vast swathes of the Northern Hemisphere. So far though, NOAA has seen no reports of major damage.

Photos: See the Northern lights from rare solar storm

The Picture Show

Photos: see the northern lights from rare, solar storm.

There has been some degradation and loss to communication systems that rely on high-frequency radio waves, NOAA told NPR, as well as some preliminary indications of irregularities in power systems.

"Simply put, the power grid operators have been busy since yesterday working to keep proper, regulated current flowing without disruption," said Shawn Dahl, service coordinator for the Boulder, Co.-based Space Weather Prediction Center at NOAA.

NOAA Issues First Severe Geomagnetic Storm Watch Since 2005

Short Wave

  • LISTEN & FOLLOW
  • Apple Podcasts
  • Google Podcasts
  • Amazon Music
  • Amazon Alexa

Your support helps make our show possible and unlocks access to our sponsor-free feed.

"Satellite operators are also busy monitoring spacecraft health due to the S1-S2 storm taking place along with the severe-extreme geomagnetic storm that continues even now," Dahl added, saying some GPS systems have struggled to lock locations and offered incorrect positions.

NOAA's GOES-16 satellite captured a flare erupting occurred around 2 p.m. EDT on May 9, 2024.

As NOAA had warned late Friday, the Earth has been experiencing a G5, or "Extreme," geomagnetic storm . It's the first G5 storm to hit the planet since 2003, when a similar event temporarily knocked out power in part of Sweden and damaged electrical transformers in South Africa.

The NOAA center predicted that this current storm could induce auroras visible as far south as Northern California and Alabama.

Extreme (G5) geomagnetic conditions have been observed! pic.twitter.com/qLsC8GbWus — NOAA Space Weather Prediction Center (@NWSSWPC) May 10, 2024

Around the world on social media, posters put up photos of bright auroras visible in Russia , Scandinavia , the United Kingdom and continental Europe . Some reported seeing the aurora as far south as Mallorca, Spain .

The source of the solar storm is a cluster of sunspots on the sun's surface that is 17 times the diameter of the Earth. The spots are filled with tangled magnetic fields that can act as slingshots, throwing huge quantities of charged particles towards our planet. These events, known as coronal mass ejections, become more common during the peak of the Sun's 11-year solar cycle.

A powerful solar storm is bringing northern lights to unusual places

Usually, they miss the Earth, but this time, NOAA says several have headed directly toward our planet, and the agency predicted that several waves of flares will continue to slam into the Earth over the next few days.

While the storm has proven to be large, predicting the effects from such incidents can be difficult, Dahl said.

Shocking problems

The most disruptive solar storm ever recorded came in 1859. Known as the "Carrington Event," it generated shimmering auroras that were visible as far south as Mexico and Hawaii. It also fried telegraph systems throughout Europe and North America.

Stronger activity on the sun could bring more displays of the northern lights in 2024

Stronger activity on the sun could bring more displays of the northern lights in 2024

While this geomagnetic storm will not be as strong, the world has grown more reliant on electronics and electrical systems. Depending on the orientation of the storm's magnetic field, it could induce unexpected electrical currents in long-distance power lines — those currents could cause safety systems to flip, triggering temporary power outages in some areas.

my cat just experienced the aurora borealis, one of the world's most radiant natural phenomena... and she doesn't care pic.twitter.com/Ee74FpWHFm — PJ (@kickthepj) May 10, 2024

The storm is also likely to disrupt the ionosphere, a section of Earth's atmosphere filled with charged particles. Some long-distance radio transmissions use the ionosphere to "bounce" signals around the globe, and those signals will likely be disrupted. The particles may also refract and otherwise scramble signals from the global positioning system, according to Rob Steenburgh, a space scientist with NOAA. Those effects can linger for a few days after the storm.

Like Dahl, Steenburgh said it's unclear just how bad the disruptions will be. While we are more dependent than ever on GPS, there are also more satellites in orbit. Moreover, the anomalies from the storm are constantly shifting through the ionosphere like ripples in a pool. "Outages, with any luck, should not be prolonged," Steenburgh said.

What Causes The Northern Lights? Scientists Finally Know For Sure

What Causes The Northern Lights? Scientists Finally Know For Sure

The radiation from the storm could have other undesirable effects. At high altitudes, it could damage satellites, while at low altitudes, it's likely to increase atmospheric drag, causing some satellites to sink toward the Earth.

The changes to orbits wreak havoc, warns Tuija Pulkkinen, chair of the department of climate and space sciences at the University of Michigan. Since the last solar maximum, companies such as SpaceX have launched thousands of satellites into low Earth orbit. Those satellites will now see their orbits unexpectedly changed.

"There's a lot of companies that haven't seen these kind of space weather effects before," she says.

The International Space Station lies within Earth's magnetosphere, so its astronauts should be mostly protected, Steenburgh says.

In a statement, NASA said that astronauts would not take additional measures to protect themselves. "NASA completed a thorough analysis of recent space weather activity and determined it posed no risk to the crew aboard the International Space Station and no additional precautionary measures are needed," the agency said late Friday.

radio waves travel at speed of sound

People visit St Mary's lighthouse in Whitley Bay to see the aurora borealis on Friday in Whitley Bay, England. Ian Forsyth/Getty Images hide caption

People visit St Mary's lighthouse in Whitley Bay to see the aurora borealis on Friday in Whitley Bay, England.

While this storm will undoubtedly keep satellite operators and utilities busy over the next few days, individuals don't really need to do much to get ready.

"As far as what the general public should be doing, hopefully they're not having to do anything," Dahl said. "Weather permitting, they may be visible again tonight." He advised that the largest problem could be a brief blackout, so keeping some flashlights and a radio handy might prove helpful.

I took these photos near Ranfurly in Central Otago, New Zealand. Anyone can use them please spread far and wide. :-) https://t.co/NUWpLiqY2S — Dr Andrew Dickson reform/ACC (@AndrewDickson13) May 10, 2024

And don't forget to go outside and look up, adds Steenburgh. This event's aurora is visible much further south than usual.

A faint aurora can be detected by a modern cell phone camera, he adds, so even if you can't see it with your eyes, try taking a photo of the sky.

The aurora "is really the gift from space weather," he says.

  • space weather
  • solar flares
  • solar storm

Researchers discover mysterious interstellar radio signal reaching Earth: 'Extraordinary'

radio waves travel at speed of sound

Mysterious radio wave pulses from deep in space have been hitting Earth for decades, but the scientists who recently discovered them have no concrete explanation for the origin of the signals.

For 35 years, the strange blasts of energy in varying levels of brightness have occurred like clockwork approximately every 20 minutes, sometimes lasting for five minute intervals. That's what Curtin University astronomers from the International Centre for Radio Astronomy Research ( ICRAR ) concluded in research published last week in the journal Nature.

The discovery of the signal, which researchers named GPMJ1839-10, has the scientists baffled. Believed to be coming from around 15,000 light years away from Earth, the signal has been occurring at intervals and for a period of time previously thought to be impossible.

“This remarkable object challenges our understanding of neutron stars and magnetars, which are some of the most exotic and extreme objects in the universe,” lead author Dr. Natasha Hurley-Walker  said in a statement on ICRAR's website.

UFO hearing: Craving more aliens? Here are 3 UFO docuseries streaming now

First signal detected from 2018 data

Using data gathered in 2018, astronomers first detected another magnetar spinning much slower than usual and sending similar signals every 18 minutes. But by the time they analyzed the data in 2020, it was no longer producing radio waves, according to Hurley-Walker.

So they looked again, knowing that the chance was high they would find another long-term radio source.

The team of astronomers used the Murchison Widefield Array radio telescope in Western Australia to scan the Milky Way galaxy every three nights for several months. They didn't have to wait long to find what they were looking for.

Within no time, a new source was discovered in a different part of the sky, this time repeating every 22 minutes with five-minute pulses.

Studying records at the Very Large Array in New Mexico, which maintains the longest-running archive of data, the researchers discovered that the source's pulse was first observed in 1988.

What's even more alarming than that this radio signal was able to go undetected for more than three decades is that scientists have not determined with confidence what it could be.

'Internet apocalypse': How NASA's solar-storm studies could help save the web

Is it a sign extraterrestrial life? Not so fast...

But before you go assuming that E.T. is trying to phone our planet, the researchers do have other theories about what may be causing it.

Even Hurley-Walker noted in an article she penned on The Conversation — a media outlet with articles written by academics and researchers — that it can be tempting to include extraterrestrial intelligence as a possible source of the signal. In fact, that's what happened when the first pulsar was discovered and astrophysicists nicknamed it "LGM 1" for "Little Green Men 1" before additional observations caused them to rule the possibility out.

The most likely culprit, researchers say, is pulsars, neutron stars that blink and rotate like lighthouses emitting energetic beams as they rotate toward and away from Earth. But pulsars slow down as time passes, their pulses growing fainter with age until they eventually stop producing radio signals.

What's more confounding: the object that the researches detected resembles a pulsar, but spins 1,000 times slower.

Another explanation researchers offer is that the object could be an ultra-long period magnetar, a rare type of neutron star with extremely strong magnetic fields that can produce powerful bursts of energy. But until recently, all known magnetars released energy at intervals ranging from a few seconds to a few minutes — far more often than the 22-minute intervals that this object emits radio waves, according to the study.

Magnetars also generate radio waves for several months before stopping, not for 35 years and counting, according to researchers. The radio emissions should be slowing down, but as observations show, it is not.

In fact, researchers note that it shouldn't be possible for it to produce radio waves at all. The object is spinning so slow as to fall below the "death line," a critical threshold where a star’s magnetic field becomes too weak to produce radio emissions.

To determine what's behind the mysterious pulsing, the astronomers said that additional observations and study are needed.

“Whatever mechanism is behind this," Hurley-Walker said in the statement, "is extraordinary.”

The discovery joins a list of mysterious finds this year beyond Earth's gravitational pull.

In May, researchers at  Sandia National Laboratories unveiled strange findings after recording unidentified sounds in the stratosphere using solar-powered balloons.

And in January, NASA's James Webb Space Telescope discovered an exoplanet outside our solar system that shares similar qualities with Earth.

Aliens among us? Vegas UFO report latest in UAP sightings investigated worldwide

Eric Lagatta covers breaking and trending news for USA TODAY. Reach him at [email protected] and follow him on Twitter @EricLagatta.

COMMENTS

  1. Radio wave

    Air is thin enough that in the Earth's atmosphere radio waves travel very close to the speed of light. The wavelength is the distance from one peak (crest) of the wave's electric field to the next, and is inversely proportional to the frequency of the wave. The relation of frequency and wavelength in a radio wave traveling in vacuum or air is.

  2. Radio Waves

    Radio waves have the longest wavelengths in the electromagnetic spectrum. They range from the length of a football to larger than our planet. Heinrich Hertz proved the existence of radio waves in the late 1880s. He used a spark gap attached to an induction coil and a separate spark gap on a receiving antenna. When waves created by the sparks of ...

  3. 5.1.1: Speeds of Different Types of Waves

    For sound waves in a fluid (for example air or water) the speed is determined by v = (B / ρ)1 / 2. v = ( B / ρ) 1 / 2. where B. B. is the bulk modulus or compressibility of the fluid in newtons per meter squared and ρ. ρ. is the density in kilograms per cubic meter. For sound waves in a solid the speed is determined by v = (Y / ρ)1 / 2.

  4. 14.1 Speed of Sound, Frequency, and Wavelength

    The amplitude of a sound wave decreases with distance from its source, because the energy of the wave is spread over a larger and larger area. But some of the energy is also absorbed by objects, such as the eardrum in Figure 14.5, and some of the energy is converted to thermal energy in the air. Figure 14.4 shows a graph of gauge pressure versus distance from the vibrating string.

  5. Understanding Radio Waves: Nature and Properties

    Understanding Radio Waves: Nature and Properties. Radio waves, the unsung heroes of the electromagnetic spectrum, serve as the cornerstone of amateur radio, enabling enthusiasts to experiment, communicate, and explore a world invisible to the naked eye. These waves, oscillating electric and magnetic fields that travel through space at the speed ...

  6. Radio propagation

    Radio propagation is the behavior of radio waves as they travel, or are propagated, from one point to another in vacuum, or into various parts of the atmosphere. [1] : 26‑1 As a form of electromagnetic radiation, like light waves, radio waves are affected by the phenomena of reflection, refraction, diffraction, absorption, polarization, and ...

  7. Electromagnetic radiation

    Radio waves are used for wireless transmission of sound messages, or information, for communication, as well as for maritime and aircraft navigation.The information is imposed on the electromagnetic carrier wave as amplitude modulation (AM) or as frequency modulation (FM) or in digital form (pulse modulation). Transmission therefore involves not a single-frequency electromagnetic wave but ...

  8. Radio wave

    The wavelengths of radio waves range from thousands of metres to 30 cm. These correspond to frequencies as low as 3 Hz and as high as 1 gigahertz (10 9 Hz). Radio-wave communications signals travel through the air in a straight line, reflect off of clouds or layers of the ionosphere, or are relayed by satellites in space.

  9. Speed of Sound (video)

    In non-humid air at 20 degrees Celsius, the speed of sound is about 343 meters per second or 767 miles per hour. We can also watch the speed of sound of a repeating simple harmonic wave. The speed of the wave can again be determined by the speed of the compressed regions as they travel through the medium.

  10. 17.1 Sound Waves

    Sound can be modeled as a pressure wave by considering the change in pressure from average pressure, ΔP = ΔPmaxsin(kx ∓ ωt + ϕ). Δ P = Δ P max sin ( k x ∓ ω t + ϕ). 17.1. This equation is similar to the periodic wave equations seen in Waves, where ΔP Δ P is the change in pressure, ΔPmax Δ P max is the maximum change in pressure ...

  11. 16.1 Traveling Waves

    Examples include gamma rays, X-rays, ultraviolet waves, visible light, infrared waves, microwaves, and radio waves. Electromagnetic waves can travel through a vacuum at the speed of light, v = c = 2.99792458 × 10 8 m/s . v = c = 2.99792458 × 10 8 m/s .

  12. Speed of Radio Waves: Physclips

    The period of the wave is measured at T = 3.3 ns (the oscillator was set at 300 MHz), so the speed is λ/T = 3.0 X 10 8 m.s −1. ( Thanks to Barry Perczuk, Pat McMillan and the UNSW third year physics lab for lending both the UHF oscillator and the 500 MHz oscilloscope. It's interesting to compare this measured speed of radio waves with the ...

  13. Physics Tutorial: The Speed of Sound

    In equation form, this is. speed = distance/time. The faster a sound wave travels, the more distance it will cover in the same period of time. If a sound wave were observed to travel a distance of 700 meters in 2 seconds, then the speed of the wave would be 350 m/s.

  14. Anatomy of an Electromagnetic Wave

    Sound waves cannot travel in the vacuum of space because there is no medium to transmit these mechanical waves. Classical waves transfer energy without transporting matter through the medium. Waves in a pond do not carry the water molecules from place to place; rather the wave's energy travels through the water, leaving the water molecules in ...

  15. Sound

    Measuring waves. All sound waves are the same: they travel through a medium by making atoms or molecules shake back and forth. But all sound waves are different too. There are loud sounds and quiet sounds, high-pitched squeaks and low-pitched rumbles, and even two instruments playing exactly the same musical note will produce sound waves that are quite different.

  16. What Is the Speed of Radio Waves? The Surprising Answer!

    Radio waves are electromagnetic radiation like sound waves, microwaves, and X-rays. All of these types of radiation travel at the same speed, which is 300,000 kilometers per second. This means that radio waves could travel around the earth seven times in a single second. It would take 8 minutes for them to travel from Earth to the Sun, and 4 ...

  17. Wavelength Calculator

    This frequency belongs to the radio waves spectrum. Choose the velocity of the wave. ... Sound in air: 343.2 m/s; Sound in water (20 °C): 1,481 m/s; ... Divide the energy by Planck's constant, 6.626 × 10⁻³⁴ J⋅Hz⁻¹, to get the frequency of the wave. Divide the speed of light, ~300,000,000 m/s, by the frequency to get wavelength.

  18. Wave Speed Calculator

    As an example, let us calculate the speed of sound waves in a medium where a 1500-Hz frequency produces a wavelength of 0.221 m. ... All electromagnetic waves, including radio waves, travel at the same speed in a vacuum. What is the SI unit of wave speed? The SI unit of wave speed is m/s.

  19. Why does it take so long for the radio waves to travel through space?

    Actually, radio waves travel very quickly through space. Radio waves are a kind of electromagnetic radiation, and thus they move at the speed of light. The speed of light is a little less than 300,000 km per second. At that speed, a beam of light could go around the Earth at the equator more then 7 times in a second. The reason that it takes so ...

  20. How Fast Do Radio Waves Travel?

    Unimpeded, radio waves travel at the speed of light because they are part of the electromagnetic spectrum. In terms of miles, radio waves travel at approximately 186,000 miles per second or 300,000,000 meters per second. If you're a science lover or just curious about the technology that makes your life easier, you've come to the right place.

  21. 15.1 The Electromagnetic Spectrum

    The narrow band of visible light is a combination of the colors of the rainbow. Figure 15.5 shows the section of the EM spectrum that includes visible light. The frequencies corresponding to these wavelengths are 4.0 × 1014 s−1 4.0 × 10 14 s −1 at the red end to 7.9 × 1014 s−1 7.9 × 10 14 s −1 at the violet end.

  22. Physics

    In air, radio waves A) travel at the same speed as sound waves B) always travel much faster than sound waves C) travel slower, on average, than sound waves. B. If a light signal and a radio signal were emitted simultaneously from Alpha Centauri, the first to reach Earth would be the A) radio signal B) light signal C) both the same time. C.

  23. Physics B

    Given that the radio waves travel at 3.00 × 108 m/s, what is the wavelength of these waves? 3.25 m. ... On a cold day, the speed of sound in air is 330 m/s. A note with a frequency of 1,320 Hz is played on an instrument. What is the wavelength of that sound wave? 0.25 m.

  24. KS1 Science: How does sound travel?

    Sound travels in waves called sound waves. Sound waves travel through particles, making them vibrate and collide with other particles. This bumping and vibrating continues, passing from particle ...

  25. The giant solar storm is having measurable effects on Earth : NPR

    Planet Earth is getting rocked by the biggest solar storm in decades - and the potential effects have those people in charge of power grids, communications systems and satellites on edge.

  26. Radio signal from space has been reaching Earth for years, study finds

    Magnetars also generate radio waves for several months before stopping, not for 35 years and counting, according to researchers. The radio emissions should be slowing down, but as observations ...